Главная Mobile Контакты NSFW Каталог Пожертвования Купить пасскод Pics Adult Pics API Архив Реквест доски Каталог стикеров Реклама
Доски

[Ответить в тред] Ответить в тред

Check this out!


<<
Назад | Вниз | Обновить тред | Автообновление
548 | 71 | 167

Тред тупых вопросов №81 Аноним 04/12/17 Пнд 10:56:14  363848  
Stephen Hawking.jpg (37Кб, 624x351)
alcubierre drive.jpg (1119Кб, 1920x1611)
Hubble Deep Fie[...].jpg (1598Кб, 2048x1536)
image.png (70Кб, 230x140)
Тред вопросов о жизни, Вселенной и всем таком.

Спрашиваем то, за что в других местах выдают путёвку в биореактор. Здесь анонимные ученые мирового уровня критически рассмотрят любые гениальные идеи и нарисованные в Paint схемы.

Q: Можно быстрее?
A: Можно упасть в пузырь альбукерке, наса уже почти надула его.

Q: Я начитался охуительных историй про уфологию, че делать, нам жопа?
A: Да, тебе жопа, можешь сгонять в зогач или куда оттуда пошлют.

Q: Что будет с человеком в вакууме без скафандра / если он упадет на черную дыру / попробует ступить на поверхность газового гиганта/солнца?
A: Он умрёт.

Предыдущий тут: >>361165 (OP)
https://2ch.hk/spc/res/361165.html#363096
Аноним 04/12/17 Пнд 14:09:36  363891
15123084054570.png (300Кб, 610x491)
Я многократно читал про гравитационные аномалии на Земле. Это и искривлённая гравитация на западе Северной Америки где-то в США на границе с Канадой, это и данные спутников об ускорении-замедлении при пролёте через определённые геологические участки, это и невозможность наблюдать человека в центре какого-то там озер, стоящего на лодке, потому что в этом месте - низина водной поверхности и в итоге получается, что для наблюдателя с берега ты сидишь как бы в овраге из воды и тебя не видно, хотя и в этот овраг, и из него ты плывёшь везде как по обычной воде.
Так вот, коли это так, можно поменять гравитацию или высчитать её и как-то целеноправленно изменить?
Аноним 04/12/17 Пнд 14:15:34  363893
>>363891
Да, можно делать концентрации вещества с большой плотностью и локально увеличивать гравитацию
Аноним 04/12/17 Пнд 14:28:51  363894
>>363320
Вот только полезный вес Союза серии МС - 100 килограмм помимо экипажа, которых не хватит даже на скафандр. И каждое снаряжение для выхода в открытый космос из БО - это адский труд, таким никто уже давным-давно не занимается и умеет только теоретически.
Аноним 04/12/17 Пнд 14:42:41  363897
>>363891
>как бы в овраге из воды и тебя не видно
Слишком охуительная история, чтобы быть правдой. Либо озеро оче большое по площади и эксперимент сугубо мысленный, пушто ИРЛ за линией горизонта не увидишь уже нихуя. Либо просто выдумки.
Аноним 04/12/17 Пнд 15:21:17  363903
>>363891
> Я многократно читал про гравитационные аномалии на Земле.
Если несёшь хуйню, то неси её с пруфами
Аноним 04/12/17 Пнд 15:30:12  363904
>>363903
https://www.youtube.com/watch?v=8ss9HYEIDvY
12:40 тут высос говорит что сие не хуйня.

Мимо другая кучка говна
Аноним 04/12/17 Пнд 15:32:33  363906
>>363904
Пардон, дважды обсерился, с 12:00, а не с 12:40
Аноним 04/12/17 Пнд 15:38:34  363907
>>363848 (OP)
Поясните за второй пик. Что отображает график? Почему посередине ровная поверхность? Если ровность поверхности означает что на корабль ничего не воздействует, то он не будет ускоряться, а будет стоять на месте. В чем тогда состоит ускоряющий эффект? Для ускорения нужен склон, по которому корабль будет "скатываться", а тут его нет. Или это график производной от ускоряющего воздействия? Если производная, то всё норм, но тогда теряется наглядность. Почему же тогда в сети часто мелькают подобные схемы?
Аноним 04/12/17 Пнд 15:42:03  363908
>>363907
Ну это же каноничный альбукерке! Сзади пространства раздвигается, спереди сжимается, по середине всё норм, чтоб корабль остался твёрдым веществом, а не кварк глюонной плазмой стал по быстрому от экстремальных перегрузок. Естественно как раздвигать, как сжимать никто не знает На астрофоруме когда то один цитатил статью, в которой говорилось что если варп и будет пахать, то световую им не превысить, если по пытаться, то распыление всей системы неминуемо
Аноним 04/12/17 Пнд 15:55:51  363909
>>363908
>Сзади пространства раздвигается, спереди сжимается, по середине всё норм
Типа двигать не корабль, а пространство? Хитро, но приемы движения пространства это слишком фантастично, может даже невозможно, потому что по краям середины будут разрывы и хрен пойми что. Не знаю, способна ли вселенная на такое насилие над ней. Гораздо лучше и понятнее ускоряться не мифическим движением пространства, а гравитационным полем. То есть корабль не стоит на месте, а ускоряется под действием гравитационного поля, которое выглядит как на картинке, только в середине склон вместо ровной поверхности, корабль просто "скатывается" по склону, все время падает в гравитационном поле. Пилоты даже перегрузок не почувствуют.
Аноним 04/12/17 Пнд 16:14:14  363911
>>363909
Сверхсвета так не выйдет полюбому, но всё равно можно было бы добиться много большего, чем обычной тягой.

>>363909
>Не знаю, способна ли вселенная на такое насилие над ней.
Тёмная энергия, вот насилует, хотя она равномерненько, а не пузырями.
Аноним 04/12/17 Пнд 16:22:48  363913
>>363903
Гравитационные аномалии - вполне рабочий термин в гравиметрии. Они под собой не подразумевают вжухающие воронки и разрывы в ткани пространства, как в сталкере. А лишь подразумевают участки, в которых картограмма демонстрирует значения измеренного ускорения свободного падения, отличающиеся от вероятного интерполируемого. Тому может быть причиной как специфика грунта, резких перепадов высот, даже атмосферы, но в таком случае точность измерений нужна очень хорошая. Гравиразведка в частности полезна в геологии. Например на основе слишком низкого значения в сравнении с ожидаемым потенциала силы тяжести можно сделать вывод, что залегающие подземные пласты состоят из вещества плотностью ниже среднего для земной коры, например природного газа. Слишком высокие значения наоборот подразумевают залежи более плотных слоев, например руд металлов. Конечно на основе только данных о потенциале силы тяжести сложно делать очевидные выводы, но можно выделить наиболее вероятные зоны, в которых уже можно проводить разведочное бурение.
Слишком выраженные глобальные аномалии, например над океанами, оно и понятно, водичка же, гораздо менее плотная, чем литосфера, уже способны влиять на траектории искусственных спутников, так что тоже учитываются. Гравиметрическая карта и для Луны составлена. Точность не абы какая, но для изучения геологии, поправок в траектории спутников годится. Офк выражения этих аномалий редко превышают тысячные доли от расчетного g. В ходу единица миллигал -мГал вроде бы. А Гал есть ускорение 1см/с2 или 1/980 от среднего значения. Поэтому истории этого>>363891
про летающие камни и дырки в озерах скорее впитанные мифы, потому что квазигеоид на человеческий взгляд, если бы его можно было охватить, поверхность все же условная, будет о-о-очень ровной поверхностью.
Аноним 04/12/17 Пнд 16:23:40  363914
>>363911
>Сверхсвета так не выйдет полюбому
Может выйдет, пока никто не пробовал же.
Аноним 04/12/17 Пнд 17:01:11  363915
Дня. А есть спутники типа Электро-Л или Himawari-8, висящие над европейской частью России? Вышеуказанные аппараты, конечно, прикольную картинку дают, но там Индийский океан да Австралия.
Аноним 04/12/17 Пнд 17:20:50  363919
>>363915
>висящие над европейской частью России?
Висеть можно только на геостационаре. То есть над экватором. Так что нет. Не было. И не будет.

Аноним 04/12/17 Пнд 18:33:56  363929
>>363919
Билят, я совсем об этом забыл. Спасибо за ответ
Аноним 04/12/17 Пнд 18:33:57  363930
Бумп
Аноним 04/12/17 Пнд 18:56:34  363931
>>363919
>Висеть можно только на геостационаре.
нет
Аноним 04/12/17 Пнд 19:10:41  363934
>>363931
Сам знаешь кого ответ.
Аноним 04/12/17 Пнд 19:50:05  363938
>>363934
можно ебашить 8-ку как на жигулях и не выходить за пределы еврорегиона
Аноним 04/12/17 Пнд 20:17:38  363939
>>363848 (OP)
Русский космос всё? Аргументируйте пожалуйста ответ.
Аноним 04/12/17 Пнд 20:26:50  363941
>>363919
>>363931
>>363938
Орбита "Молния" смеётся над вами.
Джапы себе дополнительный джипиэс запилили так.
Аноним 04/12/17 Пнд 20:28:20  363942
>>363939
Вбухали бабло в ангару - она не нужна.
Вбухали бабло в Восточный - а он без новой РН не нужен.
Старые ракеты не конкурентоспособны.
Одна зарубежная контора сделала столько же запусков, сколько весь Роскосмос.
За все время существования страны ни одного полёта в дальний космос.
Аноним 04/12/17 Пнд 20:34:54  363945
>>363942
> Одна зарубежная контора сделала столько же запусков, сколько весь Роскосмос.
За год, прошу заметить
> Старые ракеты не конкурентоспособны.
Да ты охуел
мимофорсер энергии
Аноним 04/12/17 Пнд 20:42:16  363947
>>363945
>энергии
Да ты охуел.
Сколько запусков за время существования Роскосмоса (и вообще России) она сделала?
Её не существует.
Как может несуществующая ракета быть конкурентоспособной?
Такая-то йоба была, так-то проебались, до сих пор боль от того, какое уебищное наследие вышло. Распилили движок вчетверо и поставили несколько сосисок с этими недодвижками, показательно запустили оземь и всё.
Аноним 04/12/17 Пнд 20:44:21  363948
>>363947
> колько запусков за время существования Роскосмоса (и вообще России) она сделала?
Вообще два. Ну блядь, она была примерно как СЛС по нагрузке. Как СЛС, карл. 80е года
Аноним 04/12/17 Пнд 20:45:55  363949
>>363948
>Вообще два.
В СССР. В России ни одного.
И не надо мне Энергию рекламировать, я сам хотел бы Вулкан застать и реюз боковушек.
04/12/17 Пнд 20:46:47  363951
>>363947
>недодвижками
Всё, что слабее 700 тс у Земли - не нужно?
Аноним 04/12/17 Пнд 20:50:54  363952
>>363951
Ты сам посмотри, какая хуйня вышла.
А1 слабее союза. А3 союз. А5 - слабее протона и уже нагромождение сосисок и уменьшение надежности. Конкурирует с существующими ракетами.
Если б делали на 170-х была бы йоба могущая в дальний космос.
А вот энергия бы ухх. Были же проекты Энергии-лайт, энергия классик и энергия-вулкан.
04/12/17 Пнд 20:54:28  363953
>>363949
>В СССР. В России
Энергия та же. Прогресс тот же. Энергомаш тот же. КБХА то же. НИИСК - ну, раньше назывался КБОМ. Электроприбор - это вот да, уже не то. Так о чём речь?
Аноним 04/12/17 Пнд 20:55:46  363954
>>363942
И чем всё это обернётся жля развития отечественной космической индустрии? Каковы её перспективы? Как это сможет обосновать пропаганда? Вы не думайте, это серьёзный вопрос, мне на /po/ с вашими экспертными заключениями идти.
Аноним 04/12/17 Пнд 20:56:15  363955
>>363953
>Так о чём речь?
>>363947
>Её не существует.
>Как может несуществующая ракета быть конкурентоспособной?
Вот о чём.
Аноним 04/12/17 Пнд 20:57:33  363956
image.png (172Кб, 750x560)
>>363954
Хуй знает. Или оклемаемся с Соколом на 170-х движках, спектром, луной-глоб, или не оклемаемся и окуклимся вконец. Как получится.
04/12/17 Пнд 21:00:58  363957
>>363952
>А1 слабее союза
Немного больше грузоподъёмность, чем у C-2.1в. И что?
>А3 союз
Таки вообще-то 14 т с Плесецка.
>А5 - слабее протона
Не, выводит столько же, но с Плесецка - на соответствующее наклонение, конечно же.

>Если б делали на 170-х
На одном - надо был подстроиться под уже построенный газоход.
Аноним 04/12/17 Пнд 21:03:13  363958
>>363938
>>363941
Снимки регулярные а-ля "Электро-Л" таки есть? Сдаётся мне, что на "молнии" погодные спутники не летают.
Аноним 04/12/17 Пнд 21:04:43  363960
>>363957
Суть в том, что Ангара получилась ненужной.
Мелкое говно можно и конверсионками пулять, если военам так хочется. А коммерсы с годами пуляют спутники всё жирнее и жирнее. А те, кто победнее просто попутным грузом пускают.
Если б мелкоракеты были выгодны, Илон до сих пор бы первый флакон пулял.
04/12/17 Пнд 21:14:48  363961
>>363960
>Мелкое говно можно и конверсионками пулять
Они ведь заканчиваются, и по количеству, и по сроку эксплуатации.
>коммерсы
Тут главные были военные же. Заказали РН лёгкого класса, и деньги давали только они - ну, что ж делать. А модульность в принятом виде решили использовать для этапности отработки.
Аноним 04/12/17 Пнд 21:20:21  363963
Полёты в дальний космос, к другим планетам невозможны?
Аноним 04/12/17 Пнд 21:26:59  363965
>>363963
Сейчас, при нынешних технологиях, уже возможны. Но у тебя вопрос-то не правильный. Ты бы уочнил - скорость света ли, нужно ли довозить чоловека, живым ли и т.д.
Аноним 04/12/17 Пнд 21:35:04  363967
>>363963
Возможен даже сейчас, но вопрос денег все ещё актуален.
Аноним 04/12/17 Пнд 21:35:29  363968
>>363961
>Они ведь заканчиваются, и по количеству, и по сроку эксплуатации.
Ты, часом, не хочешь сказать, что воены не делают новые ракеты?
>Тут главные были военные же.
Ну как обычно. Начали за здравие (фау-2 етц), закончили раком, убивающим космос (спесь шатал, ангара).
Аноним 04/12/17 Пнд 21:36:12  363969
>>363963
По доске прогуляйся, постоянно летают: Джуно, Кассини, Новые Горизонты итд.
04/12/17 Пнд 22:04:31  363975
>>363968
Уже давно не делают подходящие для конверсии в РН требуемой грузоподъёмности. Самым молодым УР-100Н УТТХ больше тридцати лет; Р-36М УТТХ и Р-36М2 не такие старые, но с Днепром уже все распрощались. А ракета для РС-28 только разрабатывается.
Аноним 04/12/17 Пнд 23:21:02  363977
Можно ли улететь на воздушном шаре в космос? Ясен хуй, что я имею ввиду не монголфьер с горелкой, а шар на гелии или вожороде.
Аноним 04/12/17 Пнд 23:22:34  363978
>>363977
Нет.
Аноним 05/12/17 Втр 01:18:32  363987
>>363953
Там весь состав уже обновился за 30 лет, о чем ты?
Аноним 05/12/17 Втр 05:37:21  363988
>>363914
У тебя бесконечное количество энергии есть? Значит не выйдет, скорость физически ограничена.
Аноним 05/12/17 Втр 10:58:01  363999
>>363952
Ещё ураган. Тут бы сам маск охуел, у урагана, теоретически, эффективность выше чем у флакона, так как топливо на возврат почти не нужно
Аноним 05/12/17 Втр 11:03:02  364001
Что происходит с плоскоебами, когда они видят запуск ракеты-носителя?
Аноним 05/12/17 Втр 11:05:22  364002
>>364001
Бабахают.
Аноним 05/12/17 Втр 13:01:54  364011
>>364001
Открывают для себя ось Z
Аноним 05/12/17 Втр 13:40:15  364016
Вопрос не совсем тупой но может вы ответите по умному.
МКС на орбите земли облучается солнечным излучением. С обшивки корабля выбиваются электроны из-за чего МКС заряжается положительно (фотоэлектрический эффект).
Вопросы: До какого предела может быть заряжена МКС? Что будет со спутником который подлетает к ней и потом при состыковывается?
Аноним 05/12/17 Втр 14:23:17  364017
>>364016
Фотоэлектрический эффект возникает там только в кусках кремния, сиречь в солнечных батарейках.
Аноним 05/12/17 Втр 14:54:04  364019
>>364016
Куда электроны выбьются в космосе?
Аноним 05/12/17 Втр 15:20:29  364020
>>364019
С такими вопросами пиздуй на Меркурий, Венеру и ближайший спутники Сатурна Юпитера.
Аноним 05/12/17 Втр 15:33:51  364021
>>364001
Если видят IRL: говорят, что голограмма, если на видео: CGI
Аноним 05/12/17 Втр 15:52:33  364022
>>364020
Что там?
Аноним 05/12/17 Втр 18:05:00  364032
>>364022
Можно встать на карачики и посмотреть под поверхность - ну там ещё каньон глубокий
Аноним 05/12/17 Втр 19:16:10  364042
>>363965
1.) Живым человеком запульнуть к Юпитеру, Венере, марсу и там же и остаться (ну или прилететь обратно, если не понравится)

2.) К Альфе Центавра сгонять, или где там экзопланеты земного типа понаходили, чтобы заселиться?
Аноним 05/12/17 Втр 19:20:50  364045
>>364042
1) На Марс в принципе реализуемо (хотя и не просто), даже с возвратом. На Венеру и Юпитер - только если не приземляться, а так, на орбиту выйти.
2) Даже и близко не реально пока. Долететь еще туда-сюда, но не за время жизни человека, а анабиоз пока не завезли. Возможность приземления в заранее не известных условиях на автоматике, которая мариновалась тысячи лет, тоже под вопросом.
Аноним 05/12/17 Втр 19:52:56  364048
>>364045
Короче хуйня тогда ваш космос ебучий, нахуй не нужный. Ну и зачем тогда туда деньги вбухивать? Закрыть нахуй космонавтику ебаную, открыть школ, больниц, церквей, то, что народу нужно. Нахуй не нужон космос ваш.
Аноним 05/12/17 Втр 21:09:59  364055
>>364048
> Закрыть нахуй космонавтику
И вымереть к хуям от удачно попавшего метеорита
Аноним 05/12/17 Втр 22:24:18  364057
почему не взлетел Х33?
Аноним 06/12/17 Срд 01:01:43  364063
>>364048

дак в смысле? людям нравится ракеты пулять в высоту. они даже деньги за это платят (в виде налогов). плюс там всякая полезная фигня бывает вроде предсказания погоды, геология всякая, даже из исследования других планет и спутников бывают всякие интересные и в некотором роде полезные выводы
Аноним 06/12/17 Срд 01:33:02  364069
>>363988
>скорость физически ограничена
>постулат — исходное положение какой-либо теории, принимаемое в рамках данной теории истинным без требования доказательства
Скорость искусственно ограничена в рамках теории.
Аноним 06/12/17 Срд 01:46:49  364070
hawking3.jpg (63Кб, 635x394)
>>364069
что, опять СРЁМ?
Аноним 06/12/17 Срд 02:17:42  364072
>>364057
Неудачные конструкционные решения и мудаки в руководстве. Читай отсюда и дальше >>360125
Аноним 06/12/17 Срд 02:32:41  364077
>>364069
Вам следует представить рабочую теорию, в которой с не ограничена, либо сходить на хуй.
Аноним 06/12/17 Срд 12:44:58  364094
>>364048
Ну блет ну не завезли в эту реальность КВАНТОВЫЙ ДВИГАТЕЛЬ, варп, ВАРП, гиперспейс, червоточины и прочую сверх световую парашу, до околосветовой тоже нормально не разгонишься, а если и разгоняешься то нужно че тотвроде силовых/плазменных/йоба магнитных полей чтоб не сдохнуть от набигающего потока частиц и пылинок (при околосветовой нормальной можно было бы добиться чего то вроде: через всю галактику за 7 лет, для вселенной прошло бы 100к но 7 лет реально человеком по аутировать на корабельке.) Ну может все таки и завезли и просто даже самые умные лысые обезьяны все равно слишком тупы, что бы обнаружить какие нибудь маня особенности вселенной, которые бы дали зелёный свет космической экспансии если и не на уровне Звёздного пути, то хотя бы через жопу вилкой раз раз, повторить миллиард раз и через 5 миллионов лет у человечества есть тысяча обитаемых планет.
Аноним 06/12/17 Срд 13:05:34  364095
>>364094
Есть мнение, что аналогично второму закону термодинамики, провозглашающему неуменьшение термодинамической энтропии, действует аналогичный закон насчет энтропии информационной. Т.е. чтобы узнать что-то принципиально новое сверх того, что уже известно (= локально уменьшить информационную энтропию), нужно затрачивать ебанистические усилия в сравнении с тем, как узнавалось то, что известно ранее. Возможно, по этой причине существуют даже принципиальные ограничения на познаваемость природы экспериментально-наблюдательным методом, на котором основывались все естественные науки со времен Бэкона и Галилея.
Аноним 06/12/17 Срд 13:34:10  364096
>>364095
Философская хуйня какая-то.
Аноним 06/12/17 Срд 14:27:11  364098
>>364095
Но человечество же стабильно развивается. Как только новая инфа перестаёт сыпаться на голову, происходит скачок в развитии познавательных приборов и инфы опять завались. Теперь вон и вовсе количественный анализ данных ну типа +1000 к количеству изученных звёзд на небосводе или что-то в этом духе производят компьютеры, и чем дальше, тем большие вычислительные мощности создаются. Так что "усилия" не совсем то слово, ведь количество затраченного времени больше не становится, наоборот, все открытия происходят всё быстрее и быстрее. Тут скорее чисто инженерные ограничения. Мол когда-нибудь некоторые приборы действительно достигнут совершенства и некоторые вещи мы действительно не сможем познавать и дальше. Но пока потолок не достигнут, информация даётся с примерно одинаковыми усилиями, как я это вижу. А спорим через n лет, когда все уже будут сильно грустными оттого, что нельзя заглянуть глубоко внутрь частицы и понять, струны там колеблются или что-то другое происходит, найдётся умник, который либо увеличит какую-нибудь имеющуюся частицу с помощью манипуляций с пространством или чем-то подобным, либо найдёт способ как заставить какой-нибудь крупный по меркам частиц объект вести себя как частица и благодаря этому мы сможем заглянуть на самый глубокий уровень?
Аноним 06/12/17 Срд 16:03:47  364100
>>364098
>спорим через n лет
Было бы охуенно. Только на это и надежда.

>ведь количество затраченного времени больше не становится
А вот тут я не соглашусь. Если просуммировать время и энергию, которые потратили все участники проекта JWST, то оно наверное поболее будет, чем у участников проекта БТА или, к примеру, у Галилея или Гершеля на шлифовку поверхностей своих телескопов.

>скачок в развитии познавательных приборов
Ну он тоже не на пустом месте происходит. Я не говорю, что всё пропало и, к примеру, больше 39 м рефлекторов не построят, но по мере исчерпания доступных к освоению качественно новых знаний количество прорывов в наблюдательной технике тоже будет закономерно уменьшаться.
Аноним 06/12/17 Срд 16:05:06  364101
>>364096
Высеры Пуанкаре тоже считали философской хуйней, а потом Эйнштейн создал теорию относительности.
Аноним 06/12/17 Срд 16:09:25  364102
>>364101
Тут такая антинаучная дичь, какие-то ебаные "знания" и "усилия", я просто в ахуе.
Аноним 06/12/17 Срд 16:15:04  364103
stephen-hawking[...].jpg (92Кб, 1140x660)
>>364102
Для этого треда в самый раз.
Аноним 06/12/17 Срд 19:45:45  364120
>>364100
Но Галилей наблюдал пару мыльных залуп, в то время как JWST сможет увидеть тысячи этих мыльных залуп, да ещё и у других звёзд
Аноним 06/12/17 Срд 20:22:03  364121
>>364098
> Но человечество же стабильно развивается
Я бы сказал, по экспоненте. Сравни прогресс за каждый век
Аноним 06/12/17 Срд 20:24:10  364123
>>364120
Пара мыльных залуп перевернула представление о мире. Вообще. Совсем. Полностью. Утверждение гелиоцентрической системы, открытие рельефа Луны, открытие спутников Юпитера. Вращение Солнца вокруг своей оси и еще хуеву тьму всего в физике. И все это один мужик, имея под руками хуй и пару стекляшек хуевого качества.
JWST ничего поразительно нового не откроет, не откроет ничего переворачивающего представление о мире, не откроет новое в небесной механике. Все, что он сделает - это количественно приумножит знания, может быть узнаем, что есть еще более разнообразные варианты миров, но их и без него сейчас понаоткрыто. Уэбб, если повезет, даст что-то новое в плане развития и эволюции галактик, так как инфракрасный, но опять же не сами галактики откроет. В этом плане херова тьма народу трудилась, потрачено ебовое количество ресурсов и времени, а принципиально нового, намного более глубокого понимания законов Вселенной не ожидается. Подтверждение одних гипотез, опровержение других.
ну и от кого мир охуел? От Коперника и Галилея, у которых из девайсов были подручные средства, которые сейчас доступны любому, или от невероятно сложной и дорогой машины, которую создавали годами тысячи людей?
Тоже самое с квантовой механикой что бы фактически и экспериментально обнаружить невидимые глазу частицы, которые носятся вокруг и отклоняются в магнитном поле достаточно камеры Вильсона, которую смастерить сейчас может любой идиот, а ингредиенты купить в магазине. А что бы открыть одну единственную частицу, предсказанную теоретически пришлось нескольким странам вскладчину строить невероятно сложный и самый дорогой в истории человечества механизм. Дороже только МКС.
Аноним 06/12/17 Срд 21:23:36  364127
inx960x640.jpg (52Кб, 958x640)
Косманы, почему конденсационный след от ракет закручен спиралью?
Аноним 06/12/17 Срд 21:43:27  364130
>>364127
Пассивация ступени - остатки топлива/окислителя стравливаются в касмас, а ступень понемногу закручивается из-за реактивной силы струи истекающего из неё говна (но это не точно).
06/12/17 Срд 21:45:35  364131
>>364127>>364130
Или это отделившаяся ступень МБР, закрученная после вскрытия сопел отсечки тяги.
Аноним 06/12/17 Срд 22:09:01  364135
>>364123
>JWST ничего поразительно нового не откроет, не откроет ничего переворачивающего представление о мире, не откроет новое в небесной механике. Все, что он сделает - это количественно приумножит знания, может быть узнаем, что есть еще более разнообразные варианты миров, но их и без него сейчас понаоткрыто
То же самое можно было бы сказать про Хаббл, однако ж он сильно подтолкнул и покачнул знания о вселенной.
Тот же твой аргумент можно привести и про БАК - зачем он нужен, только одну частицу открывать, вон у нас в Дубне линейный коллайдер не хуже. Дескать, только количественная экспансия.
Аноним 07/12/17 Чтв 03:47:39  364164
>>364094

джэнерейшон шипы никто не отменял. может даже высадиться на межзвёздный астероид и сделать из него дженерейшон шип, ну и лететь куда он сам летит (потому что хрен ты его орбиту изменишь). представь себе как охуенно, пара миллионов человек миллионы лет существует на (и внутри) летящей в мезвёздном пространстве горе, быдло за лоханкой сойлента обсуждает а была ли когданибудь эта самая историческая "земля" или это всё выдумки и бог создал их прямо на этой глыбе в космосе миллионы лет назад, изначально бессмертных способных жить в открытом космосе и питаться межзвездным излучением, но по мере того как они вопреки божьего запрета стремились к знаниям и технологиям бог в наказание ослабял их чтобы они больше не могли жить в космическом пространстве открытом и питаться межгалактической радиацией. и вот через миллион лет наступает момент когда они прилетают в какуюто звёздную систему и поднимают свой йобалёт размером с пол горы и в который перемещается всё население и тормозят с межзвёздной скорости в сотни километров в секунду, выходят на орбиту звезды и высаживаются на какуюнибудь из планет, это настолько охуенно что должно когданибудь реализоваться. да и вообще, так как вселенная бесконечна и в ней существует всё что только может существовать, значит гдето такой корабль уже бороздит межзвёздные просторы на протяжении миллионов лет
Аноним 07/12/17 Чтв 06:31:05  364171
1385174431193.jpg (40Кб, 400x400)
>>364123
>ну и от кого мир охуел? От Коперника и Галилея, у которых из девайсов были подручные средства, которые сейчас доступны любому, или от невероятно сложной и дорогой машины, которую создавали годами тысячи людей?
Кхм, ну если уж начать срачик и копнуть статистически и социологически, то как раз социальное устройство 15-17ых веков и нынешнее очень сильно различается. Системы иные. Вот ты говоришь про Коперника и Галилея, дескать их открытия пошатнули картину мира. Однако чью? Ватиканских попов и кучки ученых мужей Европы? Так на тот момент это была тончайшая прослойка населения. Как их публикации шатнули картину мира какого-нибудь тосканского крестьянина? Ответ очевиден. Сейчас у нас иного рода диверсификация социума. Отбросив пролетариат или средний класс, взяв интеллигенцию, тех самых физиков и лириков. Так вот меня лично выводы Коперника, Галилея и Ньютона, когда до них дошло в школьной программе, уже не шатали. Я как-то понабрался всей фактологии из детских энциклопедий и для меня их выводы были устоявшейся картиной мира. Их методы казались просто правильными и даже очевидными. Да, это их гения в моих глазах не отменяло, потому что экстраполировать можно на исторические обстоятельства, чтобы понять какое это было по сути противостояние с тяжелым укоренившимся ошибочным мнением.
Однако что меня взбудоражило действительно, так это законы Кеплера, методы наблюдения сверхновой Тихо Браге, метод измерения скорости света Олафом Ремером, уже в процессе студенческого курса по астрономии. И эти вещи как раз будоражат умы очень немногих. Большинство о них не знает, да и не нужно большинству это, функциональные задачи у людей сильно разнятся. Коперник и Галилей - просто мейнстрим. Таким ярким и громким мейнстримом от физике последний раз был Эйнштейн и его знаменитая теория. Да, с тех пор переворачивающих мировоззрение вообще всех открытий не было, но это опять с точки зрения отношения к массам. Ведь зафорсили ТО как ломающую консервативные взгляды уже после смерти Эйнштейна. И то во многом потому, что холодная война касалась на бытовом уровне всех жителей прогрессивного мира. И именно в контексте ядерной энергии/оружия она рассматривалась. Того, что касалось абсолютно всех. Сейчас у нас просто нет общемирового тренда/прецедента, который должен затронуть умы всех. Все ж науч-поп по-прежнему вещь не сказать чтобы прям массовая, хотя отголоски всех этих черных дыр и кротовых нор достигают ушей подавляющего большинства. Но для тех, кто в теме, и тем более профессионалов 20 век был просто водопадом ломающих мировоззрение открытий. Черные дыры, нейтронные звезды, эволюция звезд, сверхскопления галактик, галактические нити, закон Хаббла, реликтовое излучение, эволюция вселенной. И вглубь - деление атома, деление нуклонов, теория поля, примирение корпускулярно-волнового дуализма, вывалившееся в невероятные квантовые теории и чуть ли не магические эффекты в микромире. Развитие стандартной модели, открытие калибровочных бозонов, квантов взаимодействий. Также вспомнить прорывы в геологии, биологии и проч, которые также косвенно касаются и зависят от наблюдений планет.
Ты говоришь про одиночек, которые из говна и палок собирали свои инструменты в таком контексте, что сейчас такое будто невозможно. Так-то может быть действительно, но все же не факт. Просто сейчас такое не нужно. Да, теоретический аппарат всех наук разросся, образование стало не то, что доступным, а обязательным. Все это создает соответствующую почву, при которой безумный идейный одиночка уже не может конкурировать с научными конгломератами. А 400 лет назад вполне мог. Потому что методология была все еще под сильным влиянием религиозных догматов, которые скорее мешали.
Сейчас же и развитие информационных систем позволяет вести сотрудничество в реальном времени людей с двух противоположных сторон планеты. Но это просто оптимизация методологии. Я бы не стал это в лоб сравнивать с господствовавшим подходом середины тысячелетия. Просто обстоятельства были совершенно иными.
Аноним 07/12/17 Чтв 07:49:16  364182
>>364094
С потоками частиц все норм http://antihydrogen.livejournal.com/46578.html
Аноним 07/12/17 Чтв 11:10:00  364186
>>364123
Вот прям на 100% двачую этого анона. Сотни новых открытий ему.

>>364100-кун

Еще есть надежда, что что-то принципиально новое найдут планируемые гравитационно-волновые обсерватории типа LISA и ET:
https://en.wikipedia.org/wiki/Laser_Interferometer_Space_Antenna
https://en.wikipedia.org/wiki/Einstein_Telescope
Но вполне может статься, что они просто подтвердят существующие теории, только с охуительной точностью и на огромном числе примеров.
Аноним 07/12/17 Чтв 11:19:05  364189
>>364186
Проблема в том, что бОльшая часть крупных открытий (не очередных экзопланет, хотя подтверждение их существования в треде незаслуженно забыто), которые можно сделать просто посмотрев в телескоп уже разобрана. Увидев же какую-то непонятную ебанину, нужно садиться и долго думать, что это за хуйня такая, и, вполне вероятно, она уже была предсказана каким-нибудь упоротым физиком 20 лет назад. Низко висящие яблоки уже сорваны, теперь приходится поизворачиваться.
Аноним 07/12/17 Чтв 11:36:42  364191
>>364189
Так и я о чем. Просто помимо констатации этого факта есть еще и не совсем очевидное объяснение ему.
Аноним 07/12/17 Чтв 11:38:41  364192
>>364186
Лайза уже не сильно актуальна. Её хотели, когда от Лайго не получали результатов. Так что да, Лайза еслиитолько сможет регистрировать волны от столкновений ещё менее массивных тел и на большем расстоянии. Так-то Кип уже нобелевку отхватил за гравиволны.
Аноним 07/12/17 Чтв 11:52:42  364193
>>364192
Статью-то читал? Во-первых, она сможет слышать слияния сверхмассивных черных дыр во всей видимой Вселенной, во-вторых, она сможет находить и локализовывать цели для наблюдения LIGO за годы и месяцы до слияния, а в-третьих, просто найдет десятки тысяч двойных источников волн. Для астрофизиков LISA - это божий дар.
Аноним 07/12/17 Чтв 12:32:22  364196
>>364193
Ты туповат немного? Все, мудило, гравиволны открыты, сей бесспорно охуенный прибор будет количественно приумножать события и набирать статистику. Про слияния СМЧД это охуеть, конечно, только эти столкновения настолько редкое событие, что за время существования агрегата их будет полторы штуки. В отличии ЧД звездной и промежуточных масс и НЗ, что с успехом уже делают на LIGO. Так что иди нахуй просто и кроме букв попробуй вникнуть в смысл написанного.
Аноним 07/12/17 Чтв 12:54:02  364202
>>364196
> гравитационные волны открыты, больше в совершенно новой области астрономии ловить нечего
Аноним 07/12/17 Чтв 13:31:05  364208
>>364164
Вселенная ее бесконечна. Диаметр измерили уже – 94 миллиарда световых лет
Как ты вообще представляешь общество, способное миллионы лет жить одинаково? Попробуй найти на сраной Земляшке государство старше тысячи лет
Аноним 07/12/17 Чтв 13:35:11  364210
>>364208
Различаешь Вселенную и ее наблюдаемую часть?
Аноним 07/12/17 Чтв 13:36:46  364211
>>364208
Сан-Марино
Аноним 07/12/17 Чтв 14:16:25  364213
>>364196
>Ты туповат немного? Все, мудило, реликтовые микрорадиоволны открыты, сей бесспорно охуенный прибор будет количественно приумножать события и набирать статистику.
Аноним 07/12/17 Чтв 14:18:52  364214
>>364213
Свет вообще давно уже всем известен, нахуй еще телескопы-микроскопы какие-то?
Аноним 07/12/17 Чтв 14:27:32  364216
>>364164
Напомнило Аврору Кима Стенли Робинсона. Как раз про корабль поколений.
Почему ты термин не перевел, кстати, есть же устоявшийся перевод.
Аноним 07/12/17 Чтв 14:37:16  364217
Температура реликтового излучения сейчас крайне мала, чуть выше на пару градусов от нуля но в какое время вселенной она равнялась 100С по цельсию? Значит что в это время не было во вселенной температуры ниже этого значения?
Аноним 07/12/17 Чтв 15:16:08  364219
>>364217
в тени то было
Аноним 07/12/17 Чтв 15:33:05  364220
>>364219
Откуда тень-то?
Аноним 07/12/17 Чтв 15:41:32  364221
>>364220
От платанов.
Аноним 07/12/17 Чтв 16:20:17  364224
>>364211
Слишком маленькая хуйня, зависящая от всего мира
>>364210
У наблюдаемой диаметр ~ 26 миллиардов светолет, а вся Вселенная – 94
Аноним 07/12/17 Чтв 16:21:23  364225
>>364224
Пиздец.
Аноним 07/12/17 Чтв 19:21:06  364237
>>364217
А в какой-то момент тысячи градусов по цельсию и даже десятки тысяч.

>Значит что в это время не было во вселенной температуры ниже этого значения?
Да, значит.
Аноним 07/12/17 Чтв 19:26:24  364242
Шариков.webm (2241Кб, 712x540, 00:00:07)
>>364224
>У наблюдаемой диаметр ~ 26 миллиардов светолет, а вся Вселенная – 94
Аноним 07/12/17 Чтв 22:01:06  364276
>>364242
Сфера/Объём Хаббла.
Аноним 07/12/17 Чтв 22:11:22  364278
>>364276
Ты намекаешь, что за пределами объёма Хаббла Вселенная вдруг заканчивается или что?
Аноним 07/12/17 Чтв 22:45:03  364286
>>364278
Конечно, ей там уде некуда деваться. Только если нет плавного перехода в другую вселенную.
Аноним 07/12/17 Чтв 23:01:01  364287
>>364286
То ли ты толстый и зеленый, или просто дебил.
Аноним 07/12/17 Чтв 23:01:13  364288
>>364286
Ты же в курсе, что размер сферы Хаббла меньше, чем у наблюдаемой Вселенной?
Аноним 08/12/17 Птн 11:45:12  364315
Qsf1wpa.mp4 (9968Кб, 460x458, 00:01:34)
На новый кьюриосити такие шины поставят?
Аноним 08/12/17 Птн 13:19:13  364322
>>364315
Нет, сорт оф старые, но укрепленные, мб кто-нибудь подробнее расскажет
Аноним 08/12/17 Птн 16:06:50  364333
Есть у кого фазовая диаграмма твёрдых тяжёлых тел, типа золота, вольфрама?
Может ли в принципе быть жидкость в вакууме?
Аноним 08/12/17 Птн 17:20:59  364338
>>364333
>Может ли в принципе быть жидкость в вакууме?
Я, конечно, дурак, но почему не может быть? Реквестирую фото плавающей мочи вокруг аполлонов
Аноним 08/12/17 Птн 19:31:02  364361
Важна ли форма космического корабля для скорости его передвижения в космосе? Почему да почему нет?

Мимогуманитарий
Аноним 08/12/17 Птн 19:50:45  364364
>>364361

В зависимости от того что понимать под скоростью, космосом и космическим кораблем. На низкой орбите земли остатки атмосферы медленно тормозят (например) МКС, вынуждая тратить топливо на постоянные коррекции орбиты и стараться поворачивать станцию так что бы создавать как можно меньше сопротивления. Если гипотетически разогнать аппарат до скоростей в проценты от световой как разряженный межзвездный газ так и пылинки будет оказывать определенное влияние, разрушая корабль. По этому для движения с такими скоростями понадобятся щиты и оптимальной будет форма корабля уменьшающая сечение в направлении движения.

В общем чем менее космос пустой по отношению к скорости движения, в данном конкретном месте, тем больше форма имеет значение.
Аноним 08/12/17 Птн 20:49:45  364376
>>364364
Тупо придумал. Логично лететь конусом назад дурой вперёд. Летя носом вперёд у тебя вся площадь обшивки, кроме основания будет подвержена износу и деградации. Если основанием вперёд, то только площадь основания, которую можно сделать толще/крепче.
Аноним 08/12/17 Птн 20:56:08  364377
>>364376
>Тупо придумал.

Ты походу себе какуюто дурь вообразил. Я вообще ничего про нос-корму не писал, а уж воображать КК в роле конуса это ты вообще чисто сам придумал. Я написал что чем меньше общее сечение (в тч придуманные тобой наклонные обшивки к носу) тем лучше. Будут ли там конусы и или просто болванка впереди корабля мне без разницы, это все фантазии.


Аноним 08/12/17 Птн 21:33:03  364381
>>364376
А разве площадь сечения по наибольшей плоскости не одинаковая у конуса вне зависимости от того, как ты развернешься?

Площадь граней, обращенных к вершине, будет больше, чем у основания, да, но, например, кол-во излучения от Солнце будет такое же. Только оно будет распределено по большей площади, а значит, при однородности поверхности конуса - износ и деградация на одну и ту же единицу площади поверхности конуса будут меньше. Но лучше, наверное, обмазать УФ-стойким кремом основание конуса и развернуться им к Солнцу, так можно сэкономить на креме от загара.

А больше я с дивана никаких профитов не вижу. Греется так же, замедляется из-за солнечного ветра/остатков атмосферы одинаково, шансы поймать гаечный ключ одинаковые, износ от столкновений с межзвездной материей или как это называется на таких скоростях околонулевая.
Аноним 08/12/17 Птн 21:36:04  364383
>>364381
Вообще если конусом вперед ехать может статься, что можно экономить на защите. Миллиметра толщиной стенка наклоненной брони будет сопоставима с сантиметровой стоящей перпендикулярно, грубо говоря.
Аноним 08/12/17 Птн 21:53:30  364387
>>364377
>>Тупо придумал.

>оптимальной будет форма корабля уменьшающая сечение в направлении движения.
> Ты походу себе какуюто дурь вообразил. Я вообще ничего про нос-корму не писал, а уж воображать КК в роле конуса это ты вообще чисто сам придумал.
>Сам придумал.

Окей.
Аноним 08/12/17 Птн 22:29:19  364392
>>364315
Нитинол в качестве шин? Это шин!
Аноним 09/12/17 Суб 03:04:02  364413
>>364208

>Как ты вообще представляешь общество, способное миллионы лет жить одинаково?

ну хз а ты как представляешь? факт конечно бесспорный что всё меняется, но я считаю что ретроспектива тут не очень применима, сколько там человеческой цивилизации, ну 20-40 тысяч лет допустим максимум? это же ничтожно мало даже по сравнению с одним миллионом лет, не говорю уже про миллиарды. ну а у нас впереди как раз эти миллиарды лет, до тепловой смерти ещё очень далеко, так что както жить придётся, хочешь не хочешь, миллионы лет жить, ну в смысле человечеству жить, поколения сменять. в некотором роде мы живём в уникальном времени, как те кто ещё на том дваче сидели. мы в самом начале, а представь каково будет жить человеку когда человеческой цивилизации будет уже сотни миллионов лет? по сути по сравнению с нами живущий в 50ом миллионе лет и 60ом миллионе лет слабо отличаются, они живут примерно в одну эпоху, хотя наверняка их жизни будут колоссально отличаться (а может и нет), но всеравно живущему в 60ом миллионе лет те кто жили в 50ом миллионе гораздо ближе чем те кто жили в первом миллионе лет человеческой цивилизации и у них гораздо больше общего будет

>Почему ты термин не перевел

потому что я тупой и не придумал как перевести чтоб норм звучало
Аноним 09/12/17 Суб 08:40:40  364423
>>364338
Потому что вода при нулевом давлении превращается в пар.
Аноним 09/12/17 Суб 09:29:17  364424
image.png (103Кб, 422x962)
>>364413
>потому что я тупой и не придумал как перевести чтоб норм звучало
Держи хинт: немало терминов можно переводить таким образом.
Аноним 09/12/17 Суб 12:00:22  364429
Какие есть объективные доказательства высадки американцев на Луне, кроме фото/видео?
Знаю, что существуют образцы лунного грунта. Они там установили маяки или что-то такое? Что ещё можно в кач-ве доказательств высадки привести?
Аноним 09/12/17 Суб 12:46:14  364431
>>364429
Господи Боже мой... А ты уверен что ты сам являешься именно собой а реальность является реальностью в том смысле который мы ожидаем?
Да можно искать ЗОГоворы бесконечно и докапываться даже до чего то вроде а является ли /b/ именно БЭ или это все таки Бесконечная библиотека омнивселенских истин а рыжие молнии скрывают, ЗОГОВОР РЯЯЯЯЯЯЯ 11111111111

А вообще есть следы воздействия, есть остатки того что не увезли с собой, есть побелевший флаг, есть зеркальный отражатель, есть 1005000 гигабайт логов полёта от текстовых до видео, есть образцы и т. д, вполне достаточно что бы контрить ЗОГа.
Аноним 09/12/17 Суб 12:48:30  364432
>>364429
Хватай, листай.
http://www.skeptik.net/conspir/moonhoax.htm
https://geektimes.ru/post/253106/
и https://rationalwiki.org/wiki/Moon_landing_hoax
На рейшнлвики вообще полно хороших вещей.
Аноним 09/12/17 Суб 15:26:56  364441
>>364432
>>364431
Спасибо
Аноним 09/12/17 Суб 20:16:10  364467
6220565-2865346[...].jpg (96Кб, 650x375)
берём и делаем на орбите вокруг солнца двойную планету. не как земля-луна. а как марс-марс. вопрос вот такой - какова размера и какой траектории/орбита (и в каком количестве) могут быть спутники у каждой из этих планет, чтобы система была стабильной и спутники не улетали от своей половинки двойной планеты прочь и не падали на неё?
Аноним 09/12/17 Суб 22:55:06  364478
>>364467
Такие планеты вообще существуют? По-моему, невозможно существование такой системы, потому что притяжение звезды будет больше, и планеты не сформируются на стадии зарождения
Аноним 09/12/17 Суб 23:01:25  364479
>>364478
>Такие планеты вообще существуют?
Мы только несколько лет назад стали открывать планеты в других системах, и сильный перекос в сторону гигантских. В общем, пока в основном теории, большой наблюдательной базы нет.
Аноним 09/12/17 Суб 23:08:02  364484
>>364479
Так анон говорит о двойной планете, и чтоб масса обоих компонент была почти одинакова. Таких ещё не открыли, насколько мне известно
Аноним 09/12/17 Суб 23:12:21  364485
>>364484
Не открыли, но это ж не значит, что их нет. Зато двойных звезд просто дохуя, например. Хотя сложно представить, что такая "двойная планета" сформируется нормальным образом, можно предположить, что случайным образом планета была выброшена со своей орбиты мимогигантом типа Юпитера, а потом как-то хитро скорешилась с другой планетой. Звучит маловероятно, но звезд-то миллиарды.
Аноним 10/12/17 Вск 01:04:18  364490
>>364485
Почему тогда двойная планета невозможна, а двойных звёзд хоть жопой жуй?
Аноним 10/12/17 Вск 01:53:32  364493
>>364490
Бля, ты жопой читаешь? Я ж пишу, что скорее всего возможна, просто маловероятна. Хотя я толком обосновать не смогу, но мне кажется, что из-за комбинации размеров (масс) и расстояний, звездам особо ничего не мешает тусоваться парами -- галактика тянет довольно слабо. А вот планетам вокруг звезды мешает относительно большая гравитация звезды в центре, к которой они, по астрономическим меркам, почти впритык.
Аноним 10/12/17 Вск 03:14:35  364499
>>363848 (OP)
Сап, спейсач, думаю мой пост будет достаточно большим, и я честно говоря колебался между сци и спейс разделом, но в последнем (т.е. тут) есть раздел для глупых вопросов, да и по тематике неплохо заходит.

Принес я сюда свое мнение касательно некоторых научных теорий.

Итак, моя мысль в том, что некоторые научные теории, а точнее специальная теория относительности и корпускулярно-волновой дуализм более несостоятельны, и надо бы уже разрабатывать новые, более объемлющие теории.

В частности, мне кажется обсурдной идея с барьером в виде скорости света, типо если бы можно было что-то переместить со скоростью более скорости света, то оно попало бы в прошлое. Как вообще можно додуматься до такого, если время есть всего лишь характеристика быстроты событий, и ни в коем разе не является измерением, пригодным для перемещения?

Корпускулярно-волновой дуализм так вообще пиздец полный, из-за ебучей копенгагенской теории напридумывали такой сложной хуйни, чтобы объяснить то, чего мы не могли понять. Надо отдать должное, копенгагенская теория послужила на славу, с помощью нее мы продвинулись в микроэлектронике и прочих отраслях. Но камон, теория волны-пилота уже года 3 как получила подтверждение на макроуровне, в неквантовых системах. Вибрирующая масляная основа, на которой прыгают шарики того же масла, создавая стоячую волну - ни разу не квантовая система, но эта система полностью симулирует интерференцию в опыте Юнга, и кучу других особенностей, лишая квантовую систему своей ебучей загадочности и даруя нам знание импульса и координат частицы в любой момент времени.

В общем, писал на сонную голову, посоветуйте что-нить почитать, дабы разобраться во всем этом.
Аноним 10/12/17 Вск 06:06:12  364503
>>364499
>Как вообще можно додуматься до такого, если время есть всего лишь характеристика быстроты событий, и ни в коем разе не является измерением, пригодным для перемещения?
В этих теориях время является неотъемлемой частью пространства-времени, на котором всё и построено, привет. Взяли оттуда, что при приближении к скорости света время замедляется на определённое значение. Считая дальше, вывели, что при скорости света время всё. Проверили экспериментом. Сошлось кучу раз так-то. Что ещё надо? Бред про прошлое оправдать? Так это и не нужно, эти сказки придумывают чтобы обычный человек лучше понял, что дальше скорости света лезть низзя, ибо на самом деле фраза "быстрее скорости света" должна восприниматься такой же бредовой, как "ниже нуля по кельвину". При приближении к скорости света одно из значений тоже приближается к нулю, но никогда его не достигает, как и в случае с температурой: это скорость хода времени (для внешнего наблюдателя), поэтому и запрещают сверхсветовую скорость. На цэ могут летать только безмассовые частицы, понятно почему.
Аноним 10/12/17 Вск 09:13:55  364507
Если основную стоимость в ракетах представляют двигатели - почему бы их не спускать на землю для повторного юза? Например сверху двигателя при отстёгивании сделать парашут и небольшой двигатель, который в нужное примерно место её скинет?
Аноним 10/12/17 Вск 10:16:17  364510
>>364507
Ты только что Маск
Аноним 10/12/17 Вск 10:17:53  364512
>>364510
А почему это не сделали в союзах и др. ракетах? Вроде несложно сверхзу поставить небольшой наболдажник с парашутом и блоками управления?
И я имею ввиду не как мистер Маск, а прост ос парашута спускать вниз, а не отбрасывать как в союзе.
Аноним 10/12/17 Вск 10:23:57  364513
>>364512
Парашют не обеспечивает досточно мягкой посадки
Аноним 10/12/17 Вск 10:27:26  364515
>>364513
А если сверху поставить еще гасители или как они называются?
Аноним 10/12/17 Вск 11:39:07  364525
image.png (629Кб, 943x1386)
image.png (1194Кб, 848x1164)
>>364515
>>364512
>>364507
Ты только что блок А Энергии.
Но у Маска лучше - не надо тащить с собой парашюты и РД мягкой посадки, только ноги и рули и садиться на уже имеющиеся движки.
Аноним 10/12/17 Вск 13:13:07  364528
>>364503
Теперь ясно, но как по мне, это еще больше запутывает.
Чому просто не ввести экспоненциальный коэффициент замедления времени? Т.е. есть пространство, есть объекты, каждый из которых имеет свой коэффициент течения времени (0 < х <= 1), уменьшается, если получает ускорение по вектору движения, и увеличивается, если против.

В итоге, для стороннего наблюдателя нет скоростей, больше скорости света, а вот для человека на ракете, с ускорением, скажем, 0,01с, скорость его корабля спокойно может преодолеть скорость света и дальше линейно увеличиваться, но только потому, что его коэффициент времени постоянно замедляется.

Насколько понимаю, вроде такое объяснение там фигурирует, но в качестве увеличения массы летящего объекта, которое и замедляет время.
Аноним 10/12/17 Вск 13:57:14  364532
>>364528
А в таком случае
>есть пространство
Нет пространства. Никто не нашёл точки отсчёта. Всё движется относительно всего, нет смысла привязывать всю систему к чему-то одному.
Аноним 10/12/17 Вск 14:49:31  364538
>>364528
>но в качестве увеличения массы летящего объекта, которое и замедляет время.
Кхм, ващета нет.
Аноним 10/12/17 Вск 23:05:37  364592
>>364499
>В частности, мне кажется
Ну охуеть. Сто раз проверенные экспериментально теории, но раз тебе кажется, что хуйня, то выбрасываем, ладно.
Аноним 11/12/17 Пнд 10:47:43  364615
Каков радиус швардшильда при котором ЧД будет испарятся с той же скоростью что и поглощать материю, если учесть что ЧД окружена материей, скажем, железом в нормальных условиях?

Какова масса ЧД с диаметром 1 атома водорода?
Аноним 11/12/17 Пнд 16:44:05  364635
>>364615
rg=2Gm/c2
m = 0.67x1027 умножить на радиус атома (сам ищи)
Аноним 11/12/17 Пнд 17:43:17  364636
Есть ли не астронамически низкая вероятность, что в центри земли микроЧД?
Аноним 11/12/17 Пнд 17:52:50  364637
>>364636
>Есть ли не астронОмически низкая вероятность, что в центрЕ земли микроЧД?
Думаю, нет. Не то, чтобы это кто-то проверял, но если ьы такая была, притяжение должно было бы быть намного сильнее.
Аноним 11/12/17 Пнд 17:56:20  364638
>>364636
Нет. ЧД бы жрала планеты. Мы щас много звезд и планетных систем наблюдаем и знали бы о неконсистентном колебании звезды без видимой планеты. Плюс, сжираемая планета светилась бы. Результирующая ЧД могла бы линзировать слегонца, что тоже заметно.
Да и вообще эти спекуляции не имеют смысла, т.к. нет известного механизма образования микрочд кроме реликтовых.
Так что нет, в планете черной дыре нехуй делать.
Аноним 11/12/17 Пнд 18:52:07  364643
А почему все стартапы пилят легкие мелкоракеты вместо большой тупой ракеты?
Аноним 11/12/17 Пнд 18:53:56  364646
>>364643
Чем больше, тем сложнее и дороже.
Аноним 11/12/17 Пнд 18:55:05  364647
>>364646
Большой ТУПОЙ ракеты же.
Без отклоняемого вектора, с каким-нибудь мотором от ЗИЛа вместо ТНА и прочая.
Аноним 11/12/17 Пнд 19:15:45  364654
>>364647
Тому шо для большой ракеты нужны йоба-пихатели и прочие хитровыебанные инженерные решения, и гнилой вэвосимь из осевшего стотрицатого зилона с такими же гнилыми рессорами, не запустит большую йобу в космос.
Аноним 11/12/17 Пнд 19:18:40  364655
>>364654
Наоборот же для тупой ракеты не нужны хитрые инженерные решения.
Йоба пихатель из чугуния покрытый аблятором чтоб не городить охлаждение рубашки, нагнетание мотором со старого зила, турбина от списанного самолёта, баки со списанных вагонов-цистерн, корпус из металлолом сваренный кузмичом за пузырь.
Самое сложное тут - система управления клапанами верньеров.
Аноним 11/12/17 Пнд 19:20:45  364657
>>364643
Поиграй в КСП без РЦС и без векторным двигателем.
Аноним 11/12/17 Пнд 19:22:30  364659
>>364657
> без РЦС
Ладно, он всё равно для раков
Кто сказал, что большая тупая ракета без реактивной системы управления? Вместо того, чтобы городить сложный кардан просто вывести дополнительный выхлоп на верньеры. У нас до сих пор союзы на верньерах управляются есличо.
Аноним 11/12/17 Пнд 19:25:01  364662
>>364655
Самое сложное тут - застраховать всю эту поебень ибо удачно этот "тупой" конструктор никак не полетит.
Аноним 11/12/17 Пнд 19:27:20  364663
>>364662
Если клепать их подешевке, то и стоимость запуска сможет окупить риски. Наверное.
Си Дрегон же предлагали, но он слишком жирная йоба, ее не на всяком заводе запилишь, да и сверхтяжем он был. А я предлагаю средняк-тяж гденить на 10 тонн на НОО. Чтоб сама ракета килотонны две-три весила.
Аноним 11/12/17 Пнд 20:51:20  364674
>>364659
>Due to their weight and the extra plumbing required for their operation, vernier rockets are seldom used in new designs. Instead, as modern rocket motors became better controllable, larger thrusters could also be fired for very short pulses, resulting in the same change of momentum as a longer thrust from a smaller motor.
Аноним 11/12/17 Пнд 20:53:12  364675
>>364674
>weight and the extra plumbing
Это не проблема для большой тупой ракеты которая не гонится за массовым совершенством
>modern rocket motors became better controllable
А вот это проблемя для тупой ракеты, которая должна обладать более простыми и дешевыми агрегатами.
Аноним 11/12/17 Пнд 20:59:28  364676
Добрый день а есть ли в этом разделе обсуждение электроракетных двигателей?
Аноним 11/12/17 Пнд 21:01:16  364677
>>364676
Отдельно двигателетреда нет.
Задавай ответы здесь, тут обо всём.
Если окажется о чём-то специфическом, то тебе может понадобиться сходить в роскосмос-тред, китаетред, пускотред, космических страхов тред или вообще в омского КБ тред.
Аноним 11/12/17 Пнд 22:03:15  364680
>>364677
Да я бы сам мог ответить на пару вопросов, если это кому интересно. А так волнует вопрос согласования работы эрду с иными энергетическими системами ка.
Аноним 11/12/17 Пнд 22:09:12  364681
>>364680
А ты пиши. Никогда не знаешь, на кого наткнёшься.
Если хочешь - можешь и писать что-нибудь, вроде инфотред был (щас ссылку кинут).
Аноним 12/12/17 Втр 07:20:09  364708
>>363848 (OP)
Объясните почему звезда не нагревает всю свою систему вокруг себя (я имею ввиду ВСЁ пространство) а только объекты?

Почему если я например сейчас телепортируюсь в пространство между Меркурием и Венерой то я замёрзну а не сгорю?
Аноним 12/12/17 Втр 07:35:26  364711
>>364708
Ты сгоришь.
Аноним 12/12/17 Втр 10:16:22  364721
>>364708
Пространство нельзя нагреть. Можно нагреть материю, заполняющую пространство. В космосе материя- ужасный дефицит.
На орбите между Меркурием и Венерой ты быстро зарумянишься до хрустящей корочки.
Аноним 12/12/17 Втр 12:04:48  364732
>>364528
>с ускорением, скажем, 0.01с
Пацаны ваще ребята.
Аноним 13/12/17 Срд 09:02:35  364814
Почему все ракеты в форме колбасы? Почему нельзя их строить в форме космолетов из фантастичечких фильмов?
Аноним 13/12/17 Срд 10:57:31  364818
jackie.gif (930Кб, 336x224)
>>364814
Прост конструкторы все старперы, фантастеку смотрели годов 50-60-х, а там одна летающая колбаса. лул
Аноним 13/12/17 Срд 13:10:30  364824
>>364814
Может потому что человечеством не найдено: кристаллического топлива с сжатыми пиздаваттами энергии на которых можно газовать месяцами с удельным импульсом, как у тысяче Рапторов, а весят пару десятков КэГэ, ЙОБА систем охлаждения, которые охлаждают все и вся в сотни тысяч раз эффективнее радиаторов, ебанециумного несосаниума который в 100500 миллиардов прочнее и легче всех известных современных сплавов и даже углеродных нанотрубок, средств для создания контролируемых гравитационных и антигравитационных полей, высасывателей петаватт энергии в наносекунду из гиперпространства, матерелизатора сотен тонн антиматерии в секунду, инертного к антиматерии вещества, ну или нормальных силовых полей, неуязвимости уровня б-г для членов экипажа и бортового ИИ чтоб выдерживать овер9000 микрозивертов от всего этого йоба оборудования для манипуляций йоба энергиями ну или Овер дохренища не пропускающих радиацию материалов или опять же СИЛОВЫХ ПОЛЕЙ и т. д и т. п. А без всей этой параши очень сложно заниматься ДИЗАЙНОМ и ЭСТЕТИКОЙ
поэтому только лютый дрочь на функционализм, ради получения хоть десятых процента эффективности и только так, извиняйте.
Аноним 13/12/17 Срд 13:15:02  364825
spc7.gif (181Кб, 300x100)
>>364814
>в форме колбасы
Щито?
Аноним 13/12/17 Срд 13:51:03  364828
>>364824
>ДИЗАЙНОМ и ЭСТЕТИКОЙ
Муск вон планирует немного отдизайнить своё шипло, но это таки все равно фаллосоколбаса со стразами и особой окантовочкой так сказать, никакой сифи выпендрежчины.
Аноним 13/12/17 Срд 14:23:38  364833
>>364818
А тот, кто на пике, может задизайнить? Вроде еще не совсем старпер. Хотя вроде старше 30, хз, может и старпер.
Аноним 13/12/17 Срд 18:25:59  364851
image.png (1196Кб, 721x1200)
>>364814
>>364828
Потому, что надо пронзать атмосферу, форма дилды для пронзания наиболее оптимальна.
Если бы у нас не было атмосферы, было бы похуй, летали бы кубами - вон, лендер аполлонов как раз кубический хтонический пример.
Ракеты космос-космос (или хотя бы impact kill vehicle) тоже выглядят как кубы с соплами из всех щелей, ибо атмосферы нет.

>>364833
>не совсем старпер
>63 года
Ну охуеть теперь.
Аноним 13/12/17 Срд 19:32:11  364856
>>364851
Ну там у него морду скособочило просто, не понять.
Аноним 13/12/17 Срд 22:23:22  364874
Выше по треду мальца задели тему теоретических полетов в дальние ебеня при анабиозном сне. Чет подумалось, а это вообще имеет смысл, даже если такие технологии появятся, усыплять человека лет на триста и запускать в дальние ебеня? В этом вообще может быть смысл? О нем же все забудут уже за это время, да и мало ли, что на Земле может за это время произойти. Вот смеха то бы было, если бы человека так усыпили, запустили к ближайшей экзопланете, а когда он прилетел, то обнаружил там людей, так как за то время что он летел уже изобрели способы передвижения быстрее и опередили его с новыми экспедициями. Да и сам такой путешественник, не опустятся ли у него руки, когда он проснувшись логично осознает, что все его родственники давно мертвы и он скорее всего последний в своем роде. Наверное сколько еще закавык может возникнуть, к которым человеческое сознание может оказаться не готовым и крыша то поедет, даже при условии, что казалось бы чисто техническая сторона вопроса решена.
Аноним 14/12/17 Чтв 05:28:32  364883
>>364874
Как будто учёнишкам такие мысли не приходят. Будут готовить психологически. Домозгуются до того, что нормальный человек такого не выдержит — не будут посылать. Или будут посылать только с какими-то жёсткими условиями.
Аноним 14/12/17 Чтв 07:43:04  364884
>>364874
>не опустятся ли у него руки, когда он проснувшись логично осознает, что все его родственники давно мертвы
Давно уже придумано сирот на такие миссии посылать.
Берёшь сычинушку-корзиночку, дрессируешь, проверяешь, тестируешь и некоторые из них подходят для дальних перелётов, дай им только архив двачей и симуляторы питурда с абу запусти, чтобы было похоже на то, что он капчует не один.

>>364856
Он весь фильм такой. Годный фильм, кстати. Как Taken, только лучше.
Аноним 14/12/17 Чтв 07:56:18  364885
Если Союз отсоединившись от МКС с помощью чуда, б-жьего промысла или магии дружбы сбросит орбитальную скорость до нуля, насколько быстрее он приземлится по сравнению с обычным сходом в атмосферу?
Аноним 14/12/17 Чтв 08:55:07  364895
Какой вариант будущего вселенной является предпочтительным в современной науке?

Тепловая смерть
или
Большое сжатие, схлопывание
Аноним 14/12/17 Чтв 09:45:18  364903
>>364895
>Тепловая смерть
Это.
Сраная Вселенная расширяется с ускорением.
Аноним 14/12/17 Чтв 10:06:50  364905
>>364895
>>364903
Пытаюсь представить степень охуевания будущих живых разумных тварей, которых угораздит родиться незадолго до этого момента. Они там себе чего то наверное фантазировать будут, о смысле жизни, зачем я здесь, в чем предназначение, что такое эта самая вселенная. А тут хуякс и очередным томным пятничным вечером они узнают, что шоу "Эта вселенная" закрывается, финиталя комедия, расходитесь пожалуйста по домам ой, у вас ведь нет запасного дома.
Аноним 14/12/17 Чтв 10:25:45  364906
>>364895
>Какой вариант будущего вселенной является предпочтительным в современной науке?

Большой разрыв, либо большое сжатие.

>>364903
>Это.
Хуето. Пять раз долбоебам объяснял, что нельзя натягивать второй закон термодинамики на всю Вселенную. Блядь, даже в сраной википузии про это написано. Нет, не хочу современных представлений об устройстве мира, хочу жрать прадедушкино говно и запивать кефиром.
Аноним 14/12/17 Чтв 10:37:08  364907
>>364906
>нельзя натягивать второй закон термодинамики на всю Вселенную
Ну че ты такой категоричный? Чейта нельзя прям сразу, законом не запрещено! Другое дело, что там хронологические рамки ояебу с количеством нулей. А вот большой разрыв так самый скорый вариант, да. И был самый вероятный, пока не так давно вроде бы космологическую константу не пересчитали, и не решили, что не, дескать не канает.
Аноним 14/12/17 Чтв 10:40:32  364908
>>364906
А теперь объясняй, как большой разрыв, либо большое сжатие ты получишь с положительной космологической постоянной.

>нельзя натягивать второй закон термодинамики
А какая разница, если для рандомного обсервера это будет выглядеть именно как тепловая смерть?
Аноним 14/12/17 Чтв 18:07:08  364940
>>364874
Ну во-первых никого и не будут посылать, т.к. профита никакого. А если и пошлют ничего такого не будет. Хотя бы потому что человеку что летит поебать на такие проблемы. Вот ты говоришь, что он расстроится что за него всю работу сделают. Ему так-то наоборот хорошо. А семья не сможет умереть т.к. летит вмести с ним, ввиду того что все эти анобиозные штучки фантасты придумывают только в концепции колонизаторства. В ином случае, как я и сказал профита нет.
А забыть их не смогут. Даже если произойдёт революция в нации отправители, то та страна что будет на той территории скажет простую фразу "страна правопреемница".
А если профит исключительно только в самом факте запуска, то и на астронавта должно бы быть поебат. Самый реалистичный сценарий это отправить болванку с пластиковым маникеном, и сказать, что это путешествие в будущее.
Аноним 14/12/17 Чтв 19:15:50  364960
Фотоны это колебание струн?
Аноним 14/12/17 Чтв 19:27:08  364962
>>364960
Нет, это эманация эфира.
По мнению некоторых ВСЁ это колебания струн.
Аноним 15/12/17 Птн 09:58:22  365034
>>364960
Давай пока начнем с того, что это возбуждения фотонного поля. Подробнее здесь:
https://physics.stackexchange.com/questions/118927/if-particles-are-excitations-what-are-their-fields
>>364962
Причем "нулевые колебания".
Относительность по хардкору Аноним 15/12/17 Птн 16:44:28  365078
thats-relativit[...].webm (167Кб, 1024x576, 00:00:01)
Червотач, что почитать или посмотреть по сабжу, чтобы немного туговатый организм мог это переварить, хотя бы в общих чертах? Пробовал курить М.Гарднера и его относительность для миллионов, не осилил, сначала вроде шло, но ближе к середине книге я завис и переключился в режим японского летчика.

Может есть какая-нибудь книга с большим количеством иллюстраций для альтернативно одаренных, или какое-нибудь видео, где все это поясняется на еще более доступном уровне? Вся надежда на тебя, космоанон.
Аноним 15/12/17 Птн 23:56:02  365252
изображение.png (856Кб, 1200x623)
Что это за штуки слева и справа от ракеты?
Аноним 15/12/17 Птн 23:58:48  365253
image.png (868Кб, 1200x623)
>>365252
Не благодари.
Аноним 16/12/17 Суб 00:23:24  365262
>>365253
Я бы лизнул.
Аноним 16/12/17 Суб 00:26:51  365264
15.jpg (145Кб, 698x872)
>>365262
Ты бы и транспортно-пусковой контейнер лизнул.
Аноним 16/12/17 Суб 00:38:45  365269
>>365078
Фильм "Интерстеллар".
Аноним 16/12/17 Суб 14:44:58  365360
you-son-of-a-bi[...].webm (292Кб, 1280x720, 00:00:02)
>>365269
Остряк дохуя?
Аноним 16/12/17 Суб 17:37:00  365400
Запускали ли спутники с помощью ПГРК?
Какие вообще ракеты-конверсионки пускали и как это организовывалось? Снимают ракету из ТПК, ставят вместо боеголовки спутник с РБ, засовывают обратно в ТПК, заряжают в подлодку/шахту и пуляют как обычно?
Аноним 16/12/17 Суб 18:10:23  365414
>>365400
>ПГРК
Нет.
>Какие вообще ракеты-конверсионки пускали
Рокот, Стрела (УР-100Н), Днепр (Р-36М), Космос (Р-12), Молния (Р7/Восход), РС-12М (Тополь)

>и как это организовывалось?
Предлагаю на рынке РН с параметрами и ценой, заключают контракт и пускают, как везде. Изменяют систему управления, иногда ставят РБ. Струляют из шахты, да. Ну кроме Молнии, она, понятное дело не шахтная.
Аноним 16/12/17 Суб 18:12:32  365417
>>365414
>РС-12М (Тополь)
Ээ, так он же ПГРК.
Аноним 16/12/17 Суб 18:18:55  365419
>>365417
>Ракета 15Ж65 может эксплуатироваться в составе стационарного (15П065) или мобильного (15П165) БРК. При этом для стационарного варианта используются шахтные пусковые установки ракет, которые снимаются с вооружения или уничтожаются в соответствии с СНВ-2. Стационарную группировку создают путем переоборудования шахтных пусковых установок 15П735 и 15П718.
Аноним 16/12/17 Суб 18:20:21  365420
>>365419
Однако ж.
Спасибо.
16/12/17 Суб 18:28:01  365422
>>365400
Есть разные случаи конверсии: просто замена головной части, или добавление новых ступеней (даже без учёта РБ), причём оба варианта возможны либо с уже произведёнными ступенями, либо с новыми специально для космчисекой РН.
16/12/17 Суб 18:33:07  365424
>>365422>>365414
То есть, наверное, стоит различать переделанные снятые с эксплуатации РН и РН на базе МБР.
16/12/17 Суб 18:33:47  365425
>>365424
>снятые с эксплуатации РН
фикс - МБР, конечно же.
Аноним 16/12/17 Суб 19:06:33  365436
А почему классическую пасту "ебаный стыд, во-первых алькубьерре" удалили? Это же платина вопросотредов http://arhivach.org/thread/8170/
Аноним 16/12/17 Суб 19:20:18  365439
>>365436
Наверное она оскорбила чьи-то религиозные чувства, и тебя зарепортили.
Аноним 16/12/17 Суб 19:37:44  365441
Startmovingtola[...].jpg (18Кб, 379x263)
>>365414
>>365400
>Запускали ли спутники с помощью ПГРК?
АЛЛО
https://www.youtube.com/watch?v=QIzXI-mlJjs
https://en.wikipedia.org/wiki/Start-1
Аноним 16/12/17 Суб 19:40:56  365443
>>365441
Недурно. По идее он же так может из любой точки страны пульнуть, почему с космодр- Ах, да, районы падения.
>АЛЛО
Мамке своей аллокай, телефонист ёбаный. Что за мода сраная пошла с этими алло, блядь? Какой-то говнорепер зафорсил что ли7
Аноним 16/12/17 Суб 23:52:18  365489
а может ли быть газовый гигант по объёму больше своего солнца?
Аноним 16/12/17 Суб 23:53:58  365491
>>365489
Конечно, если его солнце белый карлик или нейтронная звезда.
Аноним 16/12/17 Суб 23:58:26  365492
>>365491
а если рассматривать не поздние этапы жизни звезды, а ранние? чтобы прямо сразу из протоплазменного диска образовалось свежее солнышко, а не прошедшая триллиард лет старуха пост-звезда
Аноним 17/12/17 Вск 00:00:14  365493
>>365492
Нит скорей всего.
Солнце будет жирнее, т.к. газики те же, а внутри термояд раздувающий оболочку. Даже если горячий юпитер, нагреваться он будет меньше, чем звезда от термояда.
Аноним 17/12/17 Вск 00:05:05  365494
>>365493
а если так: звезда сперва массы нажрала и была лёгкой, а потом нажрав массы, сколлапсировалась на саму себя, уплотнившись и начала светить иначе, но всё же молодо-зелено?
Аноним 17/12/17 Вск 00:06:31  365495
>>365494
>сколлапсировалась на саму себя
Ноуп.
Нажрав массы она просто станет больше и жить будет меньше.
Аноним 17/12/17 Вск 02:12:38  365518
>>365489
>>365491
Красные/ Коричневые карлики могут быть оче компактные и если рядом с ними газяра массой в пол Юпитера и её оче хорошо разогрела то вполне может заметно превосходить диаметр своей звезды не думаю что более чем в два раза, а скорее всего в полтора максимум
Аноним 17/12/17 Вск 02:15:14  365521
>>365518
Примерно почувствовал? Или цифры есть?
Аноним 17/12/17 Вск 03:00:33  365525
>>365521
https://www.popmech.ru/science/news-394442-otkryta-gigantskaya-planeta-kotoraya-vrashchaetsya-vokrug-ochen-malenkoy-zvezdy/
Но тут какой то жирный красный карлан, я точно помню что где то видел красный карлан 150-250 тыщь км в диаметре, а это значит планета вполне может раздуться сильнее чем звезда, коричневые карлики так вообще всегда сжаты до размеров ~100000 км диаметром, в Space Engine по крайней мере так.
Аноним 17/12/17 Вск 03:06:59  365527
>>365525
Может, эта звезда - 2MASS J0523-1403? А планета Юпитер и всё, что больше - от обычных суперюпитеров до HD 100546b
https://www.youtube.com/watch?v=GoW8Tf7hTGA
Аноним 17/12/17 Вск 15:13:16  365547
С какой целью запускают космические аппараты на ретроградные орбиты? Слышал краем уха, что подобной ерудной занимаются израильтяне, но вроде как вынужденно, чтобы не уронить на головы арабам чего-нибудь.

Есть ли какие-то профиты с такой орбиты?
Аноним 17/12/17 Вск 15:22:51  365548
>>365547
Профит разве что в том, что ты быстрее облетаешь земляшку. Профит сомнительный т.к. для разведки орбиты предпочтительны полярные.
Аноним 17/12/17 Вск 17:04:17  365559
Аноны, напомните: будет ли работать ЖРД в котором топливные компоненты подаются в камеру под действием сжатого газа, ну то есть без турбонасоса?
Аноним 17/12/17 Вск 17:11:32  365561
>>365559
Конечно, ЖРД с вытеснительной подачей давно используются - тот же AJ10 например.
Аноним 17/12/17 Вск 18:20:36  365567
>>365561
Зачем тогда у флакона баллоны с гелием, если у них турбонасосы?
17/12/17 Вск 18:22:07  365568
>>365567
Для наддува баков. Насосы тут вообще не причём.
Аноним 17/12/17 Вск 18:23:01  365569
>>365568
А никто не думал просто выдавливать баки как тюбик? Тогда и наддув не нужен и насосы.
17/12/17 Вск 18:28:56  365572
>>365569
Так тоже бывает, но у небольших баков космических аппаратов. Причём сдавливание всё равно давлением газа.
Аноним 17/12/17 Вск 18:38:24  365579
>>365569
ты сейчас сильфон придумал со стружкой
Аноним 17/12/17 Вск 22:35:28  365617
Спейсач,хелпани с миссиями апполон,когда сатурн 5 летел ,он развивал скорость больше шоб вылететь с орбиты Земли,вопрос,как он потом скорость сбавлял шоб на луну прилуниться?
17/12/17 Вск 22:42:49  365618
Saturn V никак не сбавлял, а Apollo тормозил двигателем SM для выхода на орбиту ИСЛ, позже, после разделения модулей, LM тормозил посадочным двигателем.
17/12/17 Вск 22:43:47  365619
>>365618 -> >>365617
Аноним 17/12/17 Вск 22:47:26  365620
>>365618
Благодарю
Аноним 18/12/17 Пнд 04:04:10  365653
>>365078
https://youtu.be/t0p9t7T87WM?t=3m56s
Аноним 18/12/17 Пнд 04:48:35  365654
>>364001
Говорят "выпуклая линза"
Аноним 18/12/17 Пнд 17:09:48  365708
tmb1111674049.jpg (64Кб, 905x563)
Как думаете, был ли бы смысл развивать космонавтику, если бы в Солнечной системе не было бы Луны, Марса, Юпитера, Сатурна и Меркурия? То есть была бы Земля, Венера и где-то далеко далеко Уран и Нептун. Мне кажется многим цивилизациям не везет с конфигурациями родных звездных систем и они оказываются в таких вот скучных жопах, где нет никакого стимула для развития космонавтики. Если бы не было Луны и Марса, то не было бы легких целей для первых космонавтов. Можно было бы выйти в открытый космос, как Гагарин, но следующий шаг был бы либо бессмысленным и скучным(полет к Венере), либо слишком сложным и далеким(к Урану и Нептуну), так что его всегда откладывали бы.
Нам еще повезло, мы можем с Луной играться, летать на Марс. Хоть он и скучный, дохлый, пиздец какой негостеприимный, но он есть. Если бы еще Цереры не было тоже. Вообще была бы жопа.
А кому-то может везет и у них в родной звездной системе под 3-5 планет годных для жизни с жидкой водой и они рано рано распространились. Но затем они делают вывод, что если им так легко было, то почему они не слышат других, начинают горевать думая, что они одни и не совершают следующего шага...
Нам одновременно повезло хоть с чем-то, но одновременно на не повезло как могло бы повезти.
Аноним 18/12/17 Пнд 17:11:42  365709
а еще без ранних наблюдений Луны, Марса и Юпитера наука может развивалась бы медленней...
Аноним 18/12/17 Пнд 17:29:11  365713
>>365708
>бессмысленным и скучным(полет к Венере)
Чем Венера менее осмысленна или скучна Луны и Марса? По Венере не ползают стада венероходов потому что это СЛОЖНО, а не потому что там нечего изучать в сравнении с тем же Марсом.
Хотя бы реалистичный ответ на вопрос где блядь на Венере вода, которая там должна быть, стоит не меньше чем поиски следов жидкой воды на древнем Марсе.
Аноним 18/12/17 Пнд 17:35:08  365714
>>365713
Полет к Венере это конечно круто, но все же если в наше неполноценной Солнечной Системе даже один раз слетать к Венере, то никакого следующего шага после этого сделать не получилось бы. Космонавтика должна развиваться как последовательное инкрементальное движение от одной цели к другой. У нас в нашей Солнечной системе его можно обозначить как Луна-Марс-Церера и астероиды-спутникиЮпитера. А в неполноценной Солнечной системе? Там даже Луны нет, так что какие-нибудь скептики могли бы говорить, что посадка на поверхность планеты нерациональна в ближайшем будущем и это ближайшее будущее продолжалось бы всегда.
Аноним 18/12/17 Пнд 17:51:00  365716
>>365714
>Полет к Венере это конечно круто, но все же если в наше неполноценной Солнечной Системе даже один раз слетать к Венере, то никакого следующего шага после этого сделать не получилось бы.
У нас тоже никакого следующего шага после Марса не будет, это будут просто количественные достижения.
>Космонавтика должна развиваться как последовательное инкрементальное движение от одной цели к другой
Кому должна?
>У нас в нашей Солнечной системе его можно обозначить как Луна-Марс-Церера и астероиды-спутникиЮпитера.
Ну а у них это можно обозначить Венера-Астероиды и кометы-спутники Нептуна и Урана. Отсутствие Луны можно считать как недостатком так и достоинством. Невозможен фальшстарт с флаговтыком.
Аноним 18/12/17 Пнд 17:52:13  365717
>>365713
>Хотя бы реалистичный ответ на вопрос где блядь на Венере вода, которая там должна быть, стоит не меньше чем поиски следов жидкой воды на древнем Марсе.
Уже давно обсосан этот вопрос. Воду всю сдуло солнышко, а точнее водород, который получается из за распада воды в верхних слоях атмосферы. Все из за отсутствия ЭМП. Не будь у Земляшки поля, была бы такой же токсичной высушеной каменюкой.
Аноним 18/12/17 Пнд 18:08:08  365720
>>365717
>Все из за отсутствия ЭМП.
Этот вариант уже давно спущен в унитаз по результатам фактических измерений. Темпы потери водорода Венерой на порядки ниже темпов которыми он должен выделяться из мантии. Либо что пиздит этот водород и связывает в горных породах коры, либо содержание водорода в веществе из которого образовалась Венера изначально было аномально низким.
>Не будь у Земляшки поля, была бы такой же токсичной высушеной каменюкой.
Хуита. Воду на Земляшке удерживает "холодная ловушка" в верхних слоях атмосферы, а не магнитное поле на которое все так дрочат и отчаянно пытаются пристегнуть ему какую-нибудь жизненно важную функцию(ну уникально жи, ну должно же доказывать нашу исключительность). Сказка про то что без поля мы все сдохли бы от радиации совсем уж не налазит на глобус, будет рассказывать охуительные истории с радиолизом воды.
Аноним 18/12/17 Пнд 18:28:25  365723
Анончик, расскажи плиз о квантовых эффектах - какие бывают?

Что такое квантовая запутанность как вот прямо берут пинцетиком два фотона и путают?

Как осуществляется квантовая связь? - по идее один из фотончиков должны отнести пинцетиком на другой конец планеты - на практике обычное оптоволокно

все статьи в интернете - куча рекламных общих слов нихуя конкретики и пахнет все пиздежом(((
Аноним 18/12/17 Пнд 18:47:30  365728
14307424562630.jpg (105Кб, 1281x1080)
>>365720
>Кудах-тах-тах

http://www.esa.int/Our_Activities/Space_Science/Where_did_Venus_s_water_go
Аноним 18/12/17 Пнд 18:54:49  365729
>>365708

тащемта космонавтика развивается как побочка от ракет, так что в любом случае сначала военные сделали бы МБР, а потом бы придумали концепцию военных спутников шпионов, радаров и возможно даже ударных спутников. так что спутники летали бы в любом случае даже если была бы одинокая планета-странник в межзвездном пространстве
Аноним 18/12/17 Пнд 19:22:00  365733
image.png (308Кб, 421x508)
>>365723
>расскажи плиз о квантовых эффектах - какие бывают?
Ага, блядь.
>вот прямо берут пинцетиком два фотона и путают?
Уебывают фотоном в кристалл из определенного материала, который при прохождения кристалла распадается на два, но суммарной энергией входящего. По закону сохранения импульса два получившихся фотона будут запутанными. https://arxiv.org/pdf/quant-ph/0702225.pdf
Способ, хоть и самый распространенный, но довольно топорный, есть и другие.
>квантовая связь
Под квантовой связью обычно понимают шифрование на основе квантовой запутанности, или передачи фотонов с определенной поляризацией, ключ известен только двоим, третий, решивший перехватить сообщение вмешивается в систему, проводя ее измерение, тем самым воздействуя на нее и изменяя квантовое состояние, что сразу же обнаруживается. Все это происходит через оптоволокно. Может и через пространство.
>куча рекламных общих слов нихуя конкретики и пахнет все пиздежом
Нуядаженезнаю. Квантовые сети существуют и в Китае, ив Японии, в Европе, и даже в Татарстане умеют, не говоря уже о всяких РКЦ и МГУ.
Аноним 18/12/17 Пнд 19:42:13  365735
>>365729
Спутники шпионы может быть были бы, но изыскивать средства на пилотируемые полеты в пустоту было бы сложнее. Задавались бы закономерные вопросы "а что там делать? Зачем путешествовать все дальше и дальше от одной пустой точки к другой, зачем улучшать наши возможности ракетных двигателей и дальность наших кораблей"? В нашей Солнечной системе у нас как бы есть точки назначения, destinations, которые можно посетить, создать параллельную колонию с самообеспечением.
Аноним 18/12/17 Пнд 20:00:16  365740
>>365735
Что-то мне кажется, что практически любая цивилизация даже без целей быстро смогла бы в ракеты лучше Земляшки. Люди звиздец жадные, эгоистичные, злые и всё такое. Это из-за того, что зачем-то именно сраные приматы начали обретать разумность давным-давно, причины были видимы ещё 50к лет назад. На то, как ведёт себя типичная мартышка, можно посмотреть в любом зоопарке.
Аноним 18/12/17 Пнд 21:32:31  365775
>>365723

Есть две частицы (не обязательно фотона), общий спин у них ноль. И не определён у каждой. Если одной из них присвоить -1, то у другой мгновенно станет +1 и никакие расстояния и преграды не помещают.

>>365708

Конечно был бы, потому что основные цели космонавтики как технической отрасли это строительство космических аппаратов. Большая часть используется для метеорологических, картографических и прочих наблюдений за Землёй, ещё часть для связи и навигации. Ещё некоторые аппараты для изучения неких процессов в невесомости (например МКС), ещё есть телескопы и засланцы в дальний космос. Ну и некоторая часть для контактного изучения не столь удалённых небесных тел.
Аноним 18/12/17 Пнд 21:34:02  365777
>>365775
>И не определён у каждой. Если одной из них присвоить -1, то у другой мгновенно станет +1 и никакие расстояния и преграды не помещают.
Так может он уже был -1, а у другой +1 изначально.
Как спин присваивают?
Аноним 18/12/17 Пнд 21:37:05  365778
>>365777

В поле помещают.
Аноним 18/12/17 Пнд 21:37:52  365780
>>365778
А как измеряют спин у другой частицы не влияя на него?
Аноним 18/12/17 Пнд 21:42:20  365781
>>365780

Так влияют, измерение одноразовое. То есть измерил и всё, пиздец. То есть как связь такую сделать: берётся ящик 1 и ящик 2, в обеих связанные между собой частицы, по мегобайту. И потом считывают используют по килобайту в день. И так на 1000 дней хватит. А частица связанные надо ещё очень аккуратно хранить.
Аноним 18/12/17 Пнд 21:47:46  365783
Подскажите, какова минимальная температура видимого света? 1000К? 550К? 10К? А масимальная?
Аноним 18/12/17 Пнд 21:55:11  365790
>>365783

Минимальная 700-800к, по максимуму ограничений нет, так как более горячие тела будут излучать и в оптическом диапазоне тоже. Вообще и при комнатной температуре тела светят и в оптическом диапазоне, просто интенсивность очень низкая и глазу не заметна.
https://ru.wikipedia.org/wiki/Формула_Планка
https://ru.wikipedia.org/wiki/Цветовая_температура
Аноним 18/12/17 Пнд 22:21:48  365799
Yellowstone coy[...].jpg (66Кб, 540x366)
>>365740
Опять мультиков про дружбомагию обсмотрелся?
>именно сраные приматы начали обретать разумность
Вообще эту претензию не понял. Ты бы предпочёл остаться неразумной мартышкой, как обитатели /по/? Или ящерки/рыбки/птички/цветочки/грибочки WAAAAGH с разумностью вдруг почему-то не стали бы пиздить друг друга палкой, а ходили бы все во фраках и с моноклями?
>На то, как ведёт себя типичная мартышка, можно посмотреть в любом зоопарке.
Ну это вообще пушка. Ты бы ещё Королёва или фон Брауна считал даунами из-за того, что они не умели разговаривать и срали себе в штаны - в младенческом возрасте.
Аноним 18/12/17 Пнд 22:31:49  365801
>>365790
>>365783


Вы хотите сказать, что температура пламени спички >700к?
Аноним 18/12/17 Пнд 22:38:48  365805
>>365780
Я хотел объяснить, но обнаружил, что и сам не понимаю, как именно всё это происходит. По-идее, если третья сторона вмешивается и просматривает квантово запутанный ключ, то обе стороны должны понять об этом и начинать использовать новый ключ — именно так данные и защищаются. Но как именно они должны это обнаружить и почему при проверке становится больше ошибок я не понял.
Аноним 18/12/17 Пнд 22:42:47  365806
>>365799
>ли ящерки/рыбки/птички/цветочки/грибочки WAAAAGH с разумностью вдруг почему-то не стали бы пиздить друг друга палкой, а ходили бы все во фраках и с моноклями?
Я так предполагаю. Не про все виды говорю, конечно. Ну ты посмотри на этих грязных тварей. Кидаются какашками, вредные, нехорошие и всё такое. И мы такими были когда-то, и из этих их привычек и моделей поведения и выросли все эти войны и циничность. Блжад, мы уже в космос летаем, а единого государства до сих пор нет. Ты представить себе можешь насколько это смешно?
>Ты бы ещё Королёва или фон Брауна считал даунами из-за того, что они не умели разговаривать и срали себе в штаны - в младенческом возрасте.
Я тебе не про слабоумие любого вида на ранних этапах развития. Я про модели поведения.
Аноним 18/12/17 Пнд 22:45:54  365807
>>365801

Температура на конце пламени свечи около 1900К, но свечение идёт в том числе за счёт химической реакции в продуктах сгорания.
Аноним 18/12/17 Пнд 22:57:23  365809
spsb.png (141Кб, 1075x441)
>>365790
Спасибо. Я вообще делаю симулятор эрозии интерактивный, в данный момент визуализирую свечение лавы. Добавил, как видишь келвин - ргб конвертацию с отсечкой по 700К. НО! Мое преобразование температуры в цвет рассчитано, как я понимаю, на абсолютно черное тело. А моя лава имеет emissivity 0.8 (1 - максимум). Как мне это учесть? Умножить или поделить температуру в кельвинах emissivity? (Физику совсем забыл)
Аноним 18/12/17 Пнд 23:35:40  365816
>>365806
>Кидаются какашками, вредные, нехорошие и всё такое.
Начнём с того, что мы произошли не от мартышек. Закончим тем, что экстраполировать впечатлившее тебя поведение зоопарковых мартышек даже не то что на всех приматов, а хотя бы на всех мартышек (которых овер 20 видов) - всё равно что экстраполировать поведение гопников у твоего подъезда на всё человечество.
>и из этих их привычек и моделей поведения и выросли все эти войны и циничность.
Это уже даже не пушка, это межконтинентальная баллистическая ракета с разделяющейся головной частью. Войны, циничность и всё, от чего тебе так грустно, пошло из конкуренции. Все животные конкурируют - воюют за территорию, самок, еду. Просто не все придумали автомат - наглядный пример прогресса в условиях конкуренции. Хочешь увидеть, как будет выглядеть мирная цивилизация без войн, цинизма и жадности? Посмотри на коров, которые только едят и срут, и всё.
>Я тебе не про слабоумие любого вида на ранних этапах развития. Я про модели поведения.
Ещё раз напоминаю, что мы не произошли ни от мартышек, ни даже от довольно смышлёных шимпанзе, а так вот нисхуя натягивать модели поведения с одного вида на другой - это моча в лицо сразу нескольким научным дисциплинам. Лучше смотри дальше поней.
Аноним 18/12/17 Пнд 23:40:01  365817
>>365775
ну берем два протона - (их физически брать легче) и путаем их (я правда не ебу как) и раздаем двум абонентам - в итоге имеем Сверхсветовую связь?
я правильно понял?
Аноним 18/12/17 Пнд 23:41:11  365818
>>365733
>Ага, блядь.
список эффектов слишком длинный?
или слишком сакральный?
Аноним 18/12/17 Пнд 23:46:55  365819
>>365816
Ну, я не имел ввиду прямые сравнения. Я сужу по тому, что у нас есть. >Эгоизм, войны, цинизм, недружелюбие, разрозненность — вроде бы разумны, а ведём себя хуже стайных животных, у тех инстинкты хоть как-то предполагают кооперацию со стаей, а у нас половина трясётся только за свой клочок земли и кучку напечатанных денег. Нет, у людей социум более развитый, не спорю, но блжад, стоит только рассмотреть его на более мелком уровне как пробирает грёбаный стыд.
>Посмотри на коров, которые только едят и срут, и всё.
Посмотрел. Пример-то примитивный. Я, конечно, тоже примитивные примеры приводил, но в моих опять же прослеживаются тупорылые манеры поведения орангутанг-стайл, которых у коров я как раз не наблюдаю. А я бы посмотрел на разумных коров, у которых развитие пошло от коровы до одомашнивания и меняло строение их тела эдак 100к лет.
>а так вот нисхуя натягивать модели поведения с одного вида на другой
ПРИ МА ТЫ. Приматы, знаешь? Что у приматов довольно много общего, пусть этого и совсем не хватает на видовую совместимость, знаешь? Кошка и тигр тоже не похожи. Но оба умеют красться и любят ласку (прирученные).
Аноним 19/12/17 Втр 00:06:17  365822
Можно ли увидеть фазы Юпитера или Марса? Почему?
Аноним 19/12/17 Втр 00:08:30  365823
>>365822
Можно. Потому, что они видны. И даже смотреть особо не надо - они всегда на землю освещённой стороной смотрят.
Аноним 19/12/17 Втр 05:27:44  365834
Не совсем по адресу, но всё же.
Чем на телефоне с андроидом наблюдать за вспышками иридиумов?
У меня телефон отлично поддерживает это https://play.google.com/store/apps/details?id=com.google.android.stardroid&hl=ru
Навожу на небо и вижу какие звёзды где. Но там нет спутников.
Спутники есть в платной "Star Walk 2". Скачал одну крякнутую, не показывает (думает, что не куплено) скачал другую, не запускается.
Пользуется тут кто такими штуками? Какое приложение нужно?
Аноним 19/12/17 Втр 08:23:31  365840
>>365809
>emissivity 0.8

Я не знаю что это такое.

>>365817

Да.
Аноним 19/12/17 Втр 11:35:32  365850
>>365817
Нет
Аноним 19/12/17 Втр 11:38:01  365851
>>365818
Там совсем все непросто и не интуитивно. Да, список длинный. И вряд ли тебя устроит перечисление названий, ты захочешь краткое описание, которое тебе не будет понятным, в итоге будет талмуд как на картинке к предыдущему сообщению.
Аноним 19/12/17 Втр 11:41:44  365852
>>365822
Да, но хуево.
http://crydee.sai.msu.ru/ak4/Chapt_10_133.htm
Аноним 19/12/17 Втр 11:44:16  365853
>>365840
>>emissivity 0.8
>
>Я не знаю что это такое.
Имелась ввиду светимость, или я хз как это порусски. Коэффициент отличия от абсолютно черного тела. АЧТ = 1, серое тело - меньше.

Но у меня более важный вопрос:
Правильно ли я считаю изменение температуры тела за счет излучения тепла в космос?
QChange (изменение энергии) = _StefanBoltzmannConstantpow(temperature, 4)_Emissivity -1;
temperatureChange = QChange / (_HeatCapacity
масса тела)
temperature - температура тела в кельвинах
_HeatCapacity - теплоемкость в джоуль- кельвинах
Аноним 19/12/17 Втр 11:44:25  365854
Screenshot2017-[...].png (203Кб, 720x1280)
Screenshot2017-[...].png (173Кб, 720x1280)
Screenshot2017-[...].png (81Кб, 720x1280)
>>365834
Все по адресу.
Очевидный Heavens Above очевиден.
Аноним 19/12/17 Втр 11:46:09  365855
>>365853
Разметка, что ты делаешь, прекрати
>QChange (изменение энергии) = _StefanBoltzmannConstant x temperature4
x _Emissivity -1;
>temperatureChange = QChange / (_HeatCapacity
x масса тела)
Аноним 19/12/17 Втр 11:47:02  365856
>>365855
Блэт
QChange (изменение энергии) = _StefanBoltzmannConstant x temperature4 x
_Emissivity -1;
temperatureChange = QChange / (_HeatCapacity
x масса тела)
Аноним 19/12/17 Втр 13:14:10  365864
>>365853

Я понял.
Цвет сохраняется, эммиссивити определяет насколько тело слабее светит, чем абсолютно чёрное. То есть изменяется только лишь интесивность.
Аноним 19/12/17 Втр 16:56:41  365877
>>365854
О, спасибо. То, что нужно.
Аноним 19/12/17 Втр 17:14:26  365888
>>365877
>>365854
Есть лучше, с пищалкой и кучей спутников и фильтрами по яркости.
https://www.youtube.com/watch?v=Zih6Py7zTT4
https://play.google.com/store/apps/details?id=com.runar.issdetector.pro
https://4pda.ru/forum/index.php?showtopic=285218&st=140#entry68264838
Аноним 19/12/17 Втр 17:17:06  365889
Иридиумы днём видно?
Аноним 19/12/17 Втр 17:18:15  365890
>>365889
На рассвете и закате, когда ты солнце уже не видишь, но оно ещё может отражаться от антенн спутника на тебя.
Аноним 19/12/17 Втр 17:20:47  365892
iss-detector-sa[...].png (189Кб, 540x900)
>>365888
О, оно даже погоду само знает. Показывает когда тучи, а когда ясно будет.
Аноним 19/12/17 Втр 17:45:05  365896
>>365888
>Есть лучше, с пищалкой и кучей спутников

Тащемта в хевенсе тоже можно оповещение настроить и прочие спутники он тоже покажет, если надо.
Раскраска по яркости - хуйня для имбецилов, риали, кто-то до десяти считать не умеет штоле?
Про прогноз погоды вообще молчу.
Аноним 19/12/17 Втр 17:50:27  365898
>>365896
Ты сначала установи, попробуй, а потом осуждай.
Аноним 19/12/17 Втр 17:55:46  365899
>>365898
Себе это скажи, ёбань.
Тем более, я ничего не осуждал.
Аноним 19/12/17 Втр 17:59:31  365900
>>365896
>Раскраска по яркости - хуйня для имбецилов
Она не мешает же.
>Про прогноз погоды вообще молчу.
Что тут плохого?
Аноним 19/12/17 Втр 18:27:21  365901
09db3b1730c4f9X[...].png (858Кб, 1024x683)
>>365900
В прогнозе погоды в самом по себе ничего плохого нет, но прогноз погоды:
Сильно растянут по времени, один и тот же на большую площадь земного шара, часто неточный.
При этом вспышка спутника кратка по времени и известна до секунды, видна из довольно узкого участка конкретной местности.
По этой причине смотреть на прогноз, при ловле вспышек не то что бы очень правильно. Надо смотреть на небо и в конкретном месте и непосредственно перед событием.
Аноним 19/12/17 Втр 21:42:20  365945
>Тред вопросов о жизни
Все инопланетяне, живые формы жизни, планеты и прочая поебота в фантастике и на рен тв так или иначе основывается на на том что человека окружает. Так вот мой невъебенно тупой вопрос может ли существовать то что НИКАК не основывается на земном окружении
Аноним 19/12/17 Втр 21:44:27  365946
>>365945
Упрлс?
Про другие вселенные мы рассуждать никак не можем в принципе, так что все хоть как-то имеющие право на существование организмы в этой вселенной ее законами и определяются.
Аноним 19/12/17 Втр 21:48:48  365948
>>365946
Суть не сколько в возможности существования, сколько в человеческом воображении ведь если принять теорию мультивселенной (знаю что опять таки ударяюсь в философию) то любая хуйня возможна
Аноним 19/12/17 Втр 22:10:04  365955
>>365948
Теория манявселенной просто фантазия пока не доказана.
Если веришь в манявселенные, то вопрос бессмысленен. Возможно всё! Жди, молись, постись, слушай радио Радонеж PinkiePieSwear и откроется портал в Эквестрию!
А на деле о таком рассуждать так же осмысленно, как о характеристиках орочьей шагоходной брони.
Аноним 19/12/17 Втр 23:47:13  365971
>>365955
>Теория манявселенной
гипотеза
Аноним 19/12/17 Втр 23:48:02  365972
>>365818
Ну давай я тебе три самых расхайпанных назову: эффект Казимира, Эффект Комптона, туннельный эффект. Даже я сейчас без гугла на пальцах не поясню за эффект Комптона, подзабыл. Ты же все равно, если тебе интересно, теперь пойдешь это читать куда-нибудь. Самый очевидный путь - википедия, а в ней есть целый раздел под заголовком "квантовые эффекты". Ты не мог сделать это самостоятельно?
Аноним 20/12/17 Срд 20:05:17  366073
Хм, если кровь берёт кислород из лёгких - почему не сделают йоба-насос, который бы подавал кислород в вены? Тогда можно было бы не дышать долго.
Аноним 20/12/17 Срд 20:11:35  366074
>>366073
>йоба-насос
>в вены?
Сейчас бы почитать учебник по биологии. Узнать про газообмен в легких, газовую эмболию и разницу, между артериями и венами.
Аноним 20/12/17 Срд 20:12:42  366075
>>366074
Ну в артерию, не суть же.
Аноним 20/12/17 Срд 21:08:59  366100
>>366073

Этот вопрос лучше в медаче задай.
Потому что если бы это было возможно, то в первую очередь использовалось бы в медицине, в стационарных системаж жизнеобеспечения, позволяющих жить людям без лёгких вообще.
Аноним 20/12/17 Срд 21:38:49  366102
volkano.png (264Кб, 404x399)
Повторю вопросик. Правильно ли рассчитывается изменение температуры лавы от эмиссии тепла в космос:

QChange (изменение энергии) = _StefanBoltzmannConstant x temperature4 x
_Emissivity x -1;
temperatureChange = QChange / (удельнаяТплоемкость
x масса тела)


Или масса(количество) тела тут нинужна?
Аноним 20/12/17 Срд 22:04:15  366112
>>366102
>temperatureChange = QChange / (удельнаяТплоемкость
>x масса тела)

Всё правильно.
Аноним 20/12/17 Срд 22:08:33  366115
Посоны, а почему ракеты катапультами на старте не подталкивают? Ведь смотрите: если подталкивать на старте придавая скорость в 50 (примерно) м/с катапультой, то можно залить больше топлива, так что запас тяги на старте будет не 30-40 процентов, а 15-20, и общий УИ выше.
Аноним 20/12/17 Срд 22:15:56  366116
lava.png (465Кб, 709x533)
>>366115
Легкие ракеты таки толкают, но не котопультой а пороховым зарядом. Да и то не ради экономии а что бы струей не обожгло расчет.

- Дорого\опасано. Ты себе представляешь катапульту которая 100500 тонн подбрасывает?
Аноним 20/12/17 Срд 22:28:43  366119
>>366102
>>366116
Спейсэнжинер в треде? Че это такое?
Аноним 20/12/17 Срд 22:30:38  366121
>>366116
>Ты себе представляешь катапульту которая 100500 тонн подбрасывает?

Да, я более чем представляю катапульту, которая подбрасывает на 100500 тонн, но вполне себе какой-нибудь Фэлкон или Дельту. Ничего особенного я тут не вижу.
Аноним 20/12/17 Срд 22:38:35  366125
Ayrcoffin.jpg (17Кб, 500x376)
Задача: у нас есть гроб из обедненного урана и релятивистская пушка, которая может разогнать гроб до околосветовой скорости. Как известно при разгоне до околосветовых скоростей масса объекта возрастает.
Вопрос: достигнет ли уран критической массы, если разогнать гроб до околосветовых скоростей и произойдет ли цепная ядерная реакция?
Аноним 20/12/17 Срд 22:39:29  366126
niceBottom2.png (2446Кб, 1477x827)
>>366119
Нит, это симулятор эрозии и может быть, в будущем, терраформинга

>>366121
>Да, я более чем представляю катапульту, которая подбрасывает на 100500 тонн
Нифиговая у тебя фантазия. Вояки самолет с авианосца едва-едва выталкивают не разбившись. А ты собрался 1000+ тонн подкидывать да еще и на 50 м\с

пикрандом
Аноним 20/12/17 Срд 22:40:26  366127
>>366125
> при разгоне до околосветовых скоростей масса объекта возрастает
Нет
https://elementy.ru/LHC/HEP/measures/invariant-mass
Аноним 20/12/17 Срд 22:45:46  366131
>>366127
Значит в этот гроб можно положить кого-то и выстрелить этим гробом в его родную планету, не боясь, что уран сдетонирует раньше времени?
Аноним 20/12/17 Срд 22:48:38  366133
>>366126
> симулятор эрозии и может быть, в будущем, терраформинга
Во имя чего, мистер Андерсен?
Игру пилишь?

>>366125
Ничего не случится. Но меня опередили.
На самом деле помимо "увеличения массы" на релявоскоростях есть ещё и неиллюзорное изменение времени - нейтрон может из другой галактики прилететь несмотря на время жизни в 880 секунд.
Аноним 20/12/17 Срд 22:57:09  366136
niceBottom4.png (2164Кб, 1732x756)
>>366112
>Всё правильно
Спасибо. А то я нарыл формулу без массы и охуевал.

>>366133
>Во имя чего, мистер Андерсен?
>Игру пилишь?
Прост. Люблю игры основанные на сложных моделях. До этого симулятор рыночной экономики пилил и пр. Ну то есть геймплея особо то не будет, меньше чем в sim earth

пикрандом
Аноним 20/12/17 Срд 23:00:01  366137
>>366126
>Вояки самолет с авианосца едва-едва выталкивают не разбившись.

У них жёсткие ограничения. Катапульта на авианосце паровая и за 20 минут ей можно выкинуть 24 самолёта.

>А ты собрался 1000+ тонн

Упомянутые ракеты значительно легче.
Аноним 20/12/17 Срд 23:01:07  366138
>>366125

Нет, не достигнет. Потому что масса в системе координат, связанной с гробом останется прежней.
Аноним 20/12/17 Срд 23:02:23  366139
>>366137
>>366121
>>366115
Для хоть сколь бы то ни было ощутимого прироста надо такую ебическую катапульту городить, что она вовек не окупится.
А ракеты, на минуточку, имеют обыкновение взрываться на стартовом столе распидорашивая его к хуям.
Экономии в катапульте выйдет даже меньше, чем при попытке построить космодром на эвересте.
Я даже больше скажу - экономии не выйдет вовсе, а наоборот - оба случая очень убыточны по сравнению с обычным стартом.
Аноним 20/12/17 Срд 23:24:38  366143
>>366137
>У них жёсткие ограничения. Катапульта на авианосце паровая
А ты какую предлагаешь?
Аноним 20/12/17 Срд 23:31:30  366144
NUCLEAR+PROPULS[...].jpg (117Кб, 960x720)
>>366143
Ядерную же.
мимокрокодил
Аноним 21/12/17 Чтв 00:11:13  366150
>>366139

Читай внимательнее что я предлагаю. А предлагаю я больше топлива в ракету заливать, а чтоб она не "буксовала" на старте, запускать катапультой. Кроме того, после старта ускорение ракеты около 4 м/с, то есть скорость в 50 м/с она будет набирать примерно 10с. То есть и в верхние слои атмосферы, где воздух более разряжен, ракета тоже быстрее попадёт.

>>366143

Гравитационную. Как только сила действия тела ракеты на пусковую установку падает до нуля, так груз движется вниз и толкает ракету вверз с ускорением 20-40м/с в течении 2-3 секунд.

Аноним 21/12/17 Чтв 00:12:10  366151
>>366150
*м/с/с
быстрофикс
Аноним 21/12/17 Чтв 00:31:09  366153
>>366150

По моим прикидкам, если Фэлкон
-запускать катапультой с начальной скоростью 60м/с
-увеличить массу первой ступени с 23 до 29 тонн
-увеличить запас топлива с 410 до 515 тонн

то ПН на НОО вырастет с 22 до 29.5 тонн. Это грубая прикидка, реально эффект будет выше за счёт более быстрого выхода в стратосферу, например, и более быстрого набора тяги равной тяге в пустоте.
Аноним 21/12/17 Чтв 01:24:00  366158
>>366150
Противовес в 2 массы ракеты? Слабо себе представляю, как его сначала надо где-то разогнать до 50 м/с, затем остановить после отрыва ракеты и вообще как передать такую силу на другую чашу
Аноним 21/12/17 Чтв 06:31:57  366164
>>366150
Каждый раз приходит маняоптимизатор и предлагает ракеты запускать с воздушных шаров, гор и котопульт, как какую-то йоба инновацию. При этом забывая, что ракета на преодоление силы тяжести и сопротивления воздуха тратит меньшую часть топлива, нежели на ачивмент орбитальной скорости.
Аноним 21/12/17 Чтв 11:15:46  366180
>>366136
Николас, ты ли это?
Аноним 21/12/17 Чтв 11:49:26  366186
15134568603280.jpg (15Кб, 600x337)
привет, пейсач.
у меня есть научный вопрос, основанный на ненаучных слухах. в общем, в мифах и легендах народов Земли, совсем древних, в том числе и древнее греческих и римских богов, нет-нет, да упомянается то, что луны у земли не было. точнее, не было видно на небосклоне для наблюдателя с земли. мне как человеку, любящему научный обоснуй, фантастику и тему космоса стало интересно: чем это может быть объяснимо в пределах известной сейчас науки и реалистичной твёрдой фантастики?

первый вариант: луна у земли была, просто она была намного дальше и с земли выглядела как звезда, яркая, жирная, но звезда, а не как сейчас блямбда блюдцем пересающая небо. но тогда вопрос в том, какова была её орбита?
второй вариант: луна - осколок (или вообще ядро) гипотетической планеты фаэтон, названной в честь одноимённого божества.
> Выпросил у своего отца Гелиоса позволение править солнечной колесницей, но его упряжка погубила его[5]: кони неумелого возницы отклонились от правильного направления и приблизились к земле, отчего та загорелась. Гея взмолилась к Зевсу, и тот сразил Фаэтона молнией[6], и Фаэтон рухнул в Эридан и погиб. По другому сказанию, Зевс, чтобы потушить пожар, пустил потоки (см. Всемирный потоп), и все смертные погибли, кроме Девкалиона и Пирры[7].
ну, предположить научный обоснуй с оглядкой на сайфай можно. например, та самая пятая планета действительно была, но неумелый бог-личинус расшатал орбиту и сделал её из удобной и стабильной похеренно-всратую и прокатывающуюся слишком близко ко всем другим планетам, включая землю, от приближения к которой у обоих планет по средствам электро-магнитного разгона внутреннего динамо и нагрева гравитационным трением начало припекать вулканической деятельностью и т.д. и т.п. возможно из-за этого марс умер, и венера сломалась, а гипотетический фаэтон распидорасило по всей сс превратив его обломки в луну, спутники марса, может быть пару астероидов упали на землю и венеру, может, и меркурий, часть осталась поясом астероидов, а может быть и даже стали спутником(ами) юпитера и сатурна/упали на них.
третий вариант: из-за какого-то катаклизма одно из крупных каменных тел далеко за орбитой земли врезалось в другое из крупных каменных тел за орбитой земли, из-за чего одно из этих двух тел, либо его осколок, резко изменилось в орбите и пошло в крутое пике близко к солнцу. и, сделав близкий пролёт перед солнцем, послужившим ещё и тормозом полёта, стал обессиленно и без задора отлетать обратной в дальний космос. но так как энергии уже не хватает, ещё чуть-чуть и тело бы совершило остановку и падение прямо на солнце, но тут его гравитационно перехватывает земля и всё. у нас есть луна. тогда, вероятнее всего, авария двух тел произошла где-то в районе сатурна, а его кольца - результат столкновения двух тел.

не суть, это всё если бы да кабы рассуждений на тему "как это было". как это есть сейчас - земля с луной - мы знаем. а как это было до - вот что мне интересно. то есть, насколько я понимаю, земля и без луны была вполне себе жива и обитаема. так же, видимо, без луны моря и океаны были куда спокойнее и уровень воды в приполярных областях был выше, а в экваториальных - ниже. но самое главное это вопрос электромагнитного поля. разве не должно оно без луны быть более мягким? или наоборот луна является эдаким поглотителем части потенциала поля?
Аноним 21/12/17 Чтв 11:54:09  366188
>>366158

Рычаги/блоки просто, и всё. Перед началом движения ракеты всё покоится.

>>366164

>При этом забывая, что ракета на преодоление силы тяжести и сопротивления воздуха тратит меньшую часть топлива

Ты просто думаешь что умный, а на самом деле нет.
Первая ступень фэлкона имеет характеристическую скорость примерно 3.2 км/с, при этом работает 162с, то есть за это время гравитация сообщает ракете скорость в 1.6 км/с. То есть половина всей мощи двигателей приходится на преодоление силы тяжести. Далее. На старте Фэлкон с грузом весит примерно 550т, и тягу 760т. Ускорение около3.8м/с/с. За 40с он сжигает четверть топлива первой ступени (100 тонн), весит уже 450т, и ускорение примерно 4.8. И того, за 40с со средним ускорением 4.3 ракета наберёт скорость в 180м/с и высоту в 3.4км. По грубым расчётам. Реально пикрелейтед. Не знаю, точная ли тут телеметрия, или она запаздывает, и как быстро двигатели при старте вышли на режим, но вообще всё совпадает. Так что манявсезнайка, а на деле дурачок без доступа к википедии и знаний физики за среднюю школу.
Аноним 21/12/17 Чтв 11:55:10  366189
флакон 40с.JPG (30Кб, 812x485)
>>366188
Аноним 21/12/17 Чтв 12:03:25  366191
>>366188
>гравитация сообщает ракете скорость
>1.6км/с
И? Что сказать-то хотел? Тебя для выхода на орбиту еще 6.2км/с надо.
Аноним 21/12/17 Чтв 12:06:55  366192
image.png (643Кб, 1280x720)
>>366186
Луна была задолго до человекоблядей и вообще до появления жизни на земле. Все эти мифы - хуйня и пиздёж.
Ты еще попробуй объяснить с точки зрения науки одновременное освещение Эквестрии луной и солнцем когда наступила дисгармония в начале 4 сезона.
Аноним 21/12/17 Чтв 12:12:36  366194
>>366192
не смотрю млп, потому что говно. но, в принципе, если эта луна находит очень близко к поверхности планеты и состоит из какого-то материала, обладающего способностью люминисценцировать при влиянии внешних факторов - радио-, фото-, химио- и т.д. и на твоё млп-говёном пике очевидно, что сложились условия, при которых и без того оче сильное влияние звезды наложилось на близкопланетарный полёт луны, отчего луна буквально светилась. вот только небо бы так не могло располовиниться и разукраситься - это уже художественная условная, грязный ты пониёб
Аноним 21/12/17 Чтв 12:14:57  366199
>>366191

>Тебя для выхода на орбиту еще 6.2км/с надо.

Которые сообщает вторая ступень со 100 тоннами топлива.
Аноним 21/12/17 Чтв 12:15:55  366201
b7enOba.jpg (113Кб, 1008x959)
>>366194
Правильный ответ - это просто мультик для маленьких девочек, не надо натягивать физику и логику на глобус, тем паче, что не факт, что там планета круглая.
То же самое применимо к сраным манямифам древних дегенератов - тупо маняфантазия ничего общего с реальным положением дел не имеющая. Все их боги и геркулесы одного уровня фантазия с цветными волшебными лошадьми.
Аноним 21/12/17 Чтв 12:17:18  366203
>>366201
> тем паче, что не факт, что там планета круглая.
а какая? пирамидальная? тетраэдральная?
Аноним 21/12/17 Чтв 12:18:59  366205
>>366203
Плоская, как и положено в мифах. Хотя глобус там на самом деле в школе виден - так что стоит внять совету Рика из РиМ: Don't think about it!.
Аноним 21/12/17 Чтв 12:19:32  366206
>>366199
Это замечательно, лифт который даст тебе 50-60м/с ты построишь не сможешь. Нарисуй схематично устройство и тебе итт докажут, что это на грани технической мысли и себя не окупает
Аноним 21/12/17 Чтв 12:19:34  366207
>>366201
>Все их боги и геркулесы одного уровня фантазия с цветными волшебными лошадьми.
Дешёвый полемический приём. У тебя доказательств в твоём вопросе, у него нет. Но ты себя ставишь в позицию правого, а его - заблуждающегося.
Аноним 21/12/17 Чтв 12:20:24  366208
>>366207
> *как и
inb4: быстрофикс
Аноним 21/12/17 Чтв 12:22:09  366209
>>366206
>себя не окупает
Я это давно сказал же.
Постройка обычного ебаного стола уже дорогущая штука, а чтоб еще и с лифтом, так вовсе жопень по деньгам выдет.

>>366207
Епт, это не полемика а мягкий намёк не обсуждать тупую хуйню, это даже не сайфач, где хоть какая-то опора на науку и намёк на логику, это гораздо хуже.
Аноним 21/12/17 Чтв 12:47:47  366212
Rawtemperaturev[...].png (11Кб, 730x572)
lAVATEST.png (385Кб, 1042x533)
lAVATEST2.png (195Кб, 531x463)
>>366180
Ниет, просто безработный погроммист который любит сложные модели но забыл физику.

Кстати, я сделал преобразование температуры в цвет по этому графику (пик раз) и оно работает норм.

НО! Как мне кажется, это преобразование не учитывает яркость света (которая, очевидно также зависит от температуры). на пиках 2, 3 видно что там где температура выше красного (Ю 1000 градусов) разница температуры не видна - все сливается в оранжевое пятно. Ниже 1000 градусов - я добавил линейное падение яркости до нуля в районе 600 градусов.

Вопрос - должна ли тут яркость света (не цвет) зависеть от температуры и как? (линейно или еще как)

На пиках 2 и 3
Аноним 21/12/17 Чтв 12:53:40  366213
космос0001.png (8439Кб, 2228x2392)
>>366206
> ты построишь не сможешь

Не вижу ничего сложного.
Аноним 21/12/17 Чтв 12:56:36  366214
>>366209
>Постройка обычного ебаного стола уже дорогущая штука, а чтоб еще и с лифтом, так вовсе жопень по деньгам выдет.

При такой>>366213 схеме ракета быстрее удалится от стартового стола, и воздействие раскалённых газов на его конструкции будет слабее. Так что ещё вопрос что дешевле.
Аноним 21/12/17 Чтв 13:05:22  366216
>>366214
>>366213
Ты усложнил конструкцию и добавил ещё точек отказа.
>ракета быстрее удалится от стартового стола, и воздействие раскалённых газов на его конструкции будет слабее
В воздействии выхлопа вообще мизер проблем, проблема в том, что ракеты >>366139
>А ракеты, на минуточку, имеют обыкновение взрываться на стартовом столе распидорашивая его к хуям.
Теперь ты не 10 млн у.е. тратишь на перестройку стола а 30. Охуеть профит.
Понизив при этом надежность из-за возможности отказа/заедания троса.

Хороший тупой вопрос короче. В том плане, что тупой.
В омское КБ сходи, там с единомышленниками подводные камни обсуди, кстати.
Аноним 21/12/17 Чтв 13:25:52  366219
>>366216
>Ты усложнил конструкцию и добавил ещё точек отказа.

Ну давай делать одноступенчатые пороховые ракеты с фитилём, чтоб наверняка.

>Теперь ты не 10 млн у.е. тратишь на перестройку стола а 30. Охуеть профит.

Откуда такие цифры?

>Понизив при этом надежность из-за возможности отказа/заедания троса.

Ну малыш, ну плёз. Сколько раз тросс заедает на 1000 выстрелов из арбалета, сколько на 1000 рейсов лифта, и сколько раз ракета раз взрывалась по иным причинам?

>проблема в том, что ракеты >>366139

Я уже трижды ответил на это, читай глазами.

Кстати, космодром на Эвересте даст хороший эффект, но технически это совершенно нереально. В отличии от катапульты.
Аноним 21/12/17 Чтв 13:30:50  366221
>>366219
>Ну давай делать одноступенчатые пороховые ракеты с фитилём, чтоб наверняка.
Вторая и третья ступень добавляют эффективности. Катапульта не добавляет нихуя кроме гемора.
>Откуда такие цифры?
Из жопы.
>Ну малыш, ну плёз. Сколько раз тросс заедает на 1000 выстрелов из арбалета, сколько на 1000 рейсов лифта, и сколько раз ракета раз взрывалась по иным причинам?
Ты, блжад, палец с жопой не сравнивай. Аргумент уровня "ну у бати лада 20 лет без поломок ездила, хули протон упал?". Конструкция манякатапульты для ракеты в разы сложнее сраного лифта. Во многие многие разы.
>Я уже трижды ответил на это, читай глазами.
В каком треде? В этом в твоём единственном ответе ни слова про взрыв на старте.
>Кстати, космодром на Эвересте даст хороший эффект, но технически это совершенно нереально. В отличии от катапульты.
Зато у них у обоих есть кое-что общее: оба маняпроекты которые никаких профитов не несут и никогда не будут претворены в жизнь.
Аноним 21/12/17 Чтв 13:37:41  366222
>>366221
>Катапульта не добавляет нихуя кроме гемора.

Я трижды рассказал почему прибавляет.

>Из жопы.

Да я заметил.

> Конструкция манякатапульты для ракеты в разы сложнее сраного лифта.

Та же синхронизация замков, троссы и это всё. Краны в портах таскают тяжёлые грузы на троссах и ничего, норм.

>В этом в твоём единственном ответе ни слова про взрыв на старте.

Почему? Слова есть, ты почему-то решил что на старте обязательно всё должно взорваться, почему я так и не понял. Наверно опять жопа тебе подсказала.
Аноним 21/12/17 Чтв 13:53:54  366223
>>366186
>луна у земли была, просто она была намного дальше
Наоборот, Луна была гораздо ближе. Это установленный наукой факт. Мимо.
>луна - осколок
Луна не осколок, Луна состоит примерно из тех же изотопов, что и Земля, их количественно-качественные характеристики очень схожи. Что делает невозможным принимать теорию ебеневого зарождения Луны.
>планеты фаэтон
Никогда не существовала.
>одно из крупных каменных тел далеко за орбитой земли
Смотри выше.

Аноним 21/12/17 Чтв 13:54:48  366224
image.png (821Кб, 900x611)
image.png (29Кб, 649x657)
>>366222
>Я трижды рассказал почему прибавляет.
У тебя там какой-то десяток м/с. Ёбаный жалкий десяток.
Ты понимаешь, что это НИХУЯ не добавляет кроме гемора?
Даже если у тебя на выходе ракета будет иметь 100м/с это все равно хуйня, т.к. она поднимется на жалкие полкилометра без двигателя - это эквивалентно старту с полукилометровой высоты.
Напомнить, что даже с Эвереста если стартовать профиты будут хуёвые, даже если забить на логистику и прочий связанный гемор?
Напомнить, что даже воздушный старт с 10км и +300м/с боковой скорости не несёт особо профитов?
А ты предлагаешь в РАЗЫ усложнить стартовый стол ради профита который нивелируется усложнением конструкций?

>Краны в портах таскают тяжёлые грузы на троссах и ничего, норм.
Краны не разгоняют сотни тонн с ускорением в полтора же.
А для ракеты надо конструкцию размерами и сложностью с крупнейшие экскаваторы. Эти йобы стоят сотни миллионов долларов, а если подобную хрень делать space grade то будет ещё дороже.
>Почему? Слова есть, ты почему-то решил что на старте обязательно всё должно взорваться, почему я так и не понял. Наверно опять жопа тебе подсказала.
Ни слова от тебя про то, что делать когда ракета ёбнет.
А жопа подсказала: ракеты взрываются. Потому, что они, чёрт побери, взрываются, или ты с этим собрался спорить т.к. в твоём мирке любая техника безупречна и непогрешима?

Советую тебе немножко посидеть с сопроматом и посчитать эти свои тросы и шкивы для буста ракеты, посчитать сколько будет стоить соорудить такую йобу чтобы получались твои циферки.
И напомню одну простую вещь, если у тебя еще не было сопромата (или даже если был): если простой анон до чего-то может додуматься, то уж ясен хуй до этого додумались ученые и инженеры, которые этим занимаются профессионально, и если они не стали так делать, то скорей всего ты просто что-то не знаешь.
Ты напомнил меня в детстве когда я придумал нереактивный движитель который имел грузики на концах палок, вертел ими, потом подтягивал их через центр назад и опять "грёб". Про сохранение импульса забыл, потом бил себя в лоб от такой очевидной промашки. А вот недавно именно такую хуйню запустили в космос и закономерно обосрались.
Аноним 21/12/17 Чтв 14:03:37  366225
>>366224
>У тебя там какой-то десяток м/с. Ёбаный жалкий десяток.
>Даже если у тебя на выходе ракета будет иметь 100м/с это все равно хуйня, т.к. она поднимется на жалкие полкилометра без двигателя - это эквивалентно старту с полукилометровой высоты.

Я трижды рассказывал, а ты ни разу не прочитал.

>А для ракеты надо конструкцию размерами и сложностью с крупнейшие экскаваторы.

Ничё подобного, у экскаватора куча систем кроме троссов.

>Ни слова от тебя про то, что делать когда ракета ёбнет.

Конечно. Ещё про то, как её собирать, как заправлять, и как разделять ступени тоже ни слова. Потому что ответ банален: то же, что и всегда в таких случаях.

>Советую тебе немножко посидеть с сопроматом

Весьма глупо слушать советы по изучению чего-либо от анона, который не может прочитать написанное мной ИТТ в трёх местах.
Аноним 21/12/17 Чтв 14:05:04  366226
>>366225
Да читал я, ты хуйню пишешь тупую. Говно твоя идея, и уже не раз рассказал почему.
Иди в омское КБ, там картинками обменивайтесь, тут ты поддержки не получишь.
Аноним 21/12/17 Чтв 14:09:03  366230
>>366226
>Да читал я

Чем, интересно, раз пишешь

>У тебя там какой-то десяток м/с. Ёбаный жалкий десяток.
>это эквивалентно старту с полукилометровой высоты
>Из жопы.

Сфоткай эти органы, плиз, и выложи сюда. Мне прям любопытно.
Аноним 21/12/17 Чтв 14:10:41  366231
>>366226
>тут ты поддержки не получишь

Типичная черта стукачей, пиздаболов, пидорах, интриганов — говорить за кого-то другого.
Аноним 21/12/17 Чтв 14:11:54  366232
>>366230
>>366231
У меня от твоей настойчивости и семенства испанский стыд.
Иди в этот тред уже >>359504 (OP)
А лучше в сайфач вообще.
Аноним 21/12/17 Чтв 14:17:12  366235
>>366213
Как ты ускорение в 30м/с собрался получить? G = 9.78 ,хоть противовес в 100 масс используй
Аноним 21/12/17 Чтв 14:28:56  366241
>>366231
Я - другой хуй и тоже уринирую твою гортань тугой струей.
Аноним 21/12/17 Чтв 14:29:33  366242
>>366235

Ну ёбаный ты даун, рычагом блядь. РЫ-ЧА-ГОМ. Рычагом, полиспасом, эксцентриком. Я тут перед публикой распинаюсь как ракеты запускать, а они не понимают как требушет устроен и одновременно отправляют сопромат учить.
Аноним 21/12/17 Чтв 15:40:50  366250
>>366242
Ещё раз: требушетом ракеты в омском КБ запускай.
Или посчитай сопромат и покажи выкладки с пруфами, что не омск.
Аноним 21/12/17 Чтв 16:07:03  366252
>>366250

Спешите видеть: гумус, узнавший про правило рычага ИТТ требует расчётов с пруфами.
Аноним 21/12/17 Чтв 16:22:38  366255
mentosnasa.jpg (198Кб, 960x946)
>>366252
Ты какой-то долбоеб, честное слово.
Уровня пикрелейтед.
Аноним 21/12/17 Чтв 16:31:22  366256
>>366252
При чём тут правило рычага?
Ты может этим рычагом землю перевернёшь?
Или в духе ТТВ вообще возьмешь длинную палку в несколько световых лет и толкнув ее передашь инфу выше скорости света?
Аноним 21/12/17 Чтв 16:32:41  366257
>>366252
Дебил, это >>366235 мой пост и я ни на что не претендую. Развел тут срач, пиздец
Аноним 21/12/17 Чтв 16:35:28  366260
>>366257
>>366235
Гм, вообще-то я хоть и против нашего требушетника, но рычагом ты вполне можешь увеличить ускорение.
Ускорение - результат приложения силы. Сила пропорциональна рычагу.
Если на рычаге длиной 1 дать 1 ньютон, то на другом конце в 0.1 расстоянии будет 10 ньютон, а на расстоянии 10 будет 0.1.
Аноним 21/12/17 Чтв 16:50:56  366267
>>366255

Зашивайся.

>>366256

Ты троллишь тупостью или что?

Ещё раз, даём ракете стартовый пинок в 60м/с, запас топлива в ракете увеличиваем на 15 процентов, двигатели оставляем прежними.
Аноним 21/12/17 Чтв 17:53:11  366273
Каким примерно должен быть Хаббл2 чтобы получить годное фото в 1к с планеты у Проксимы Центавра
Аноним 21/12/17 Чтв 17:57:44  366274
>>366273

Боюсь, что единственный способ раглядеть на таком расстоянии поверхность планеты, это отправить туда спутник. Слишком сильные искажения на таких дистанциях, волна расходится.
Аноним 21/12/17 Чтв 18:17:54  366277
>>366274
Какие у тебя искажения в открытом космосе? Какая нахуй волна там расходится? Все упирается исключительно в апертуру.

>>366273
>Каким примерно должен быть Хаббл2 чтобы получить годное фото в 1к с планеты у Проксимы Центавра
Что за 1к? Это что за характеристика?
Что за характеристика "годное фото"?
Либо строить зеркало в несколько сот метров, либо делать систему оптических интеоферометров с базой в несколько сот, а лучше в несколько тысяч километров.
Аноним 21/12/17 Чтв 18:20:23  366279
>>366277
>Какая нахуй волна там расходится?

Электромагнитная. А что? Это ты тут выше не понимал как гравитационной катапультой ускорение выше g получить?
Аноним 21/12/17 Чтв 18:46:10  366280
>>366279
Ты знаешь, что такое эта самая электромагнитная волна, дебила кусок?
На телескопах получают изображения более крупных объектов, которые гораздо дальше и ничего. А с ростом апертуры ебать магия растёт разрешение, представляешь? Нет никаких физических ограничений в получении изображения, есть технологические. Если ты нихуя не допетриваешь в оптике, так ебальник не раскрывай и не позорься.

Аноним 21/12/17 Чтв 18:55:48  366282
>>366280
Как насчет гравитационных искажений?
мимо
Аноним 21/12/17 Чтв 19:07:42  366283
>>366280
>Ты знаешь, что такое эта самая электромагнитная волна

А ты знаешь?

>На телескопах получают изображения более крупных объектов

Даже без телескопов получают изображения более крупных объектов, дальше что?

Пока я вижу воинствующего невежу, который рвётся и горит.
Аноним 21/12/17 Чтв 19:11:34  366285
>>366280
>Нет никаких физических ограничений в получении изображения, есть технологические.

Ну да, всего-то построить хаббл-2 с в сто раз бОльшим зеркалом. Так нет и физических ограничений на межзвёздное путешествие, сделать 100-ступенчатую ракету с начальной массой как у планеты, и всё.

>>366283-кун
Аноним 21/12/17 Чтв 19:45:27  366291
worst post ever.png (419Кб, 803x688)
>>366267
>даём ракете стартовый пинок в 60м/с, запас топлива в ракете увеличиваем на 15 процентов
Нет, правда. В омское КБ.
Аноним 21/12/17 Чтв 21:12:58  366307
>>366291

Я, кстати, посчитал уже точно. Прибавка получается крайне внушительная: 137км вместо 112 и 2350м/с вместо 2280.
Аноним 21/12/17 Чтв 21:56:25  366310
>>366282
Какие гравитационные искажения мешают разглядывать, например, галактику Андромеды через тот же Хаббл?
Да что там гравитационные, мутную атмосфера уже не такая большая помеха для адаптивной оптики.

>>366283
>дальше что?
Ничего, можешь пойти нахуй, если туповат и не соображаешь про что вообще речь идет.

>>366285
>Ну да, всего-то построить хаббл-2 с в сто раз бОльшим зеркалом.
А ты решил прикинуться долбоебом? Анон задал абстрактный вопрос.
>Так нет и физических ограничений на межзвёздное путешествие, планеты, и всё.
Да, представляешь. Межзвездные путешествия вполне возможны, нет ни одного закона природы их запрещающие. или ебучий плоскоземелец и у тебя небесная твердь?
>сделать 100-ступенчатую ракету с начальной массой как у Земли
Вообще-то нужна намного более скромная ебулда, но ты продолжай кривляться, ебанашка.
Аноним 21/12/17 Чтв 22:05:02  366311
Вы ебнутые? Идите курите угловой размер
Аноним 21/12/17 Чтв 23:22:10  366315
>>366310
>Ничего, можешь пойти нахуй, если туповат и не соображаешь про что вообще речь идет.

По существу ничего, ясно.

>А ты решил прикинуться долбоебом? Анон задал абстрактный вопрос.

Он спросил про хаббл-2.

>Вообще-то нужна намного более скромная ебулда, но ты продолжай кривляться, ебанашка.

Пруф.

>>366311

Да я-то в курсе угловых размеров, но даже просто по угловым размерам 10км на Проксиме Центавре это как 1мм на поверхности Луны (если с Земли смотреть). А именно о таком разрешении идёт речь. Но дело ещё в соотношении неопределённости.
Аноним 21/12/17 Чтв 23:23:23  366316
>>366310
>Вообще-то нужна намного более скромная ебулда, но ты продолжай кривляться, ебанашка.

Ну давай, считай какая нужна ракета чтоб добраться до ближайшей звезды хотя бы за сто лет. Нихуя ты не посчитаешь, ты ведь даже правило рычага не знаешь, гумуса кусок.
Аноним 21/12/17 Чтв 23:24:54  366317
Неужели эту ракету, запущенную с помощь манякатапульты не распидорасит к хуям от перегрузки, так как воздействие катапульты будет очевидно коротким по времени, и поэтому придется придавать ей охуенное ускорение чтобы хоть как-то значимо успеть разогнать?
Аноним 21/12/17 Чтв 23:41:09  366320
>>366317

Ускорение при воздействии катапульты 3..4g. По силе примерно как воздействие боковушек в фэлкон хеви/дельта хеви только меньше по времени (2-3с).
Аноним 21/12/17 Чтв 23:53:15  366321
Скажите пожалуйста, а при квантовой связи вообще никаких волн не должно быть? и кроме расстояния ничего не мешает передаче данных?
или такую связь еще не создали и это ляля?
Ученые уже поняли ЧТО именно удерживает в связи спутанные частицы? или это из области сакральной магии - работает и пиздец!?
Аноним 22/12/17 Птн 01:13:43  366325
fukken lold7.jpg (137Кб, 980x1040)
>>366307
>Прибавка получается крайне внушительная: 137км вместо 112 и 2350м/с вместо 2280.
Цифры в студию!
Аноним 22/12/17 Птн 01:15:30  366326
>>366317
>Неужели эту ракету, запущенную с помощь манякатапульты не распидорасит к хуям от перегрузки, так как воздействие катапульты будет очевидно коротким по времени, и поэтому придется придавать ей охуенное ускорение чтобы хоть как-то значимо успеть разогнать?
Во-первых, нужна ебическая катапульта по габаритам чтобы хоть как-то ощутимый прирост дать, во-вторых она будет стоит в разы дороже ракет и будет распидорашена к хуям первой же зафейлевшей ракетой.
Аноним 22/12/17 Птн 07:53:26  366337
>>366326

Оля-ля, здесь мы идём.

>>366325

Да ладно зачем же тебе цифры, ты же гуманитарий. Нет ну правда
>ебическая
>разы дороже
>распидорашена к хуям

Ну зачем тебе количественные характеристики, что ты будешь с ними делать?
Аноним 22/12/17 Птн 08:38:25  366341
>>366320
Интересно, что за чудо-агрегат передаст такое усилие на бандуру несколько сотен тонн весом
>>366326 правильно говорит
Аноним 22/12/17 Птн 09:23:54  366345
Теперь манякатапульта и правило рычага станут локальными мемасами итотэ треда.
Аноним 22/12/17 Птн 09:25:34  366346
>>366320
Тогда 2-3с при 3-4g она увеличит скорость ракеты примерно на 60-120м\с? Ну охуеть какой прирост!
Аноним 22/12/17 Птн 12:01:06  366355
>тросс
>тросс
Помимо этого
>не учитывать инерцию подвижных частей
Не помимо, а совершенно точно вливаю мочу и кладу кал в твой рот.
Аноним 22/12/17 Птн 12:20:47  366356
>>366345
>локальными мемасами итотэ треда.
Да, и Омич-полуёбок, отлично будет смотреться в одном ряду с почившими Рогулем и Антисети-петухом.
Аноним 22/12/17 Птн 13:54:03  366364
>>366341

ИТТ всё есть.

>>366346

При стартовом ускорении в 60м/с к концу работы первой ступени 270м/с дельты.

>>366355

Гумус порвался в клочья, найс.
Аноним 22/12/17 Птн 13:55:14  366365
>>366345

Ну вот, видишь. Откуда бы ты узнал про правило рычага, если бы не я. Так глядишь и до второго закона Ньютона дорастёшь.
Аноним 22/12/17 Птн 14:06:27  366366
>>366365
Принимай таблетки и уебывай к омичам в тренд.
Аноним 23/12/17 Суб 00:04:46  366423
521723postmedia[...].jpg (32Кб, 1024x714)
Первым объектом на гелиоцентрической орбите стал аппарат Луна-1. И где он сейчас? Кто-нибудь нашел его на радаре, знает его орбиту?
Аноним 23/12/17 Суб 00:05:46  366424
>>366423
Хз))0)
Аноним 23/12/17 Суб 00:08:36  366427
>>366423
Такую мелкую хуёвину разве возможно отследить?
Аноним 23/12/17 Суб 00:22:49  366431
>>366423
>Кто-нибудь нашел его на радаре,
Она слишком мелкая, мы вряд ли сможем её отследить на нынешнем технологическом уровне. Мы даже куда более крупный лунный модуль Snoopy на гелиоцентрической не можем найти.
>знает его орбиту
Мы знаем, что она по идее должна быть на эллиптической орбите 146 млн. x 197 млн. километров (то есть она пересекается с земной), и совершает один виток вокруг Солнца примерно за 450 дней. Где конкретно на этой орбите - не знаем, смотри выше. Не исключено, что на одном из витков Земля выкинула Луну-1 нахуй куда-нибудь подальше.
Аноним 23/12/17 Суб 00:47:59  366438
Еще один вопрос к шизику. Какие материалы будут использованы при изготовлении твоей поеботины, чтобы выдержать усилие на разрыв\изгиб?
Аноним 23/12/17 Суб 02:06:51  366444
>>366423

>нашел его на радаре

я уже задавал этот вопрос в этом треде, мне тупому объяснили почему радар не работает в космосе - электромагнитное излучение рассеивается-слабеет с квадратом расстояния. тоесть чтобы обнаружить самолет в 100км достаточно бортового источника питания самолета, а вот чтобы засечь радаром чтото в сотнях миллионов километров от земли нужна просто ебическая мощность излучателя. успех предприятия ещё зависит от ширины луча. если у тебя излучающая и принимающая антенны радара с очень узким лучом то тебе понадобится гораздо меньше энергии чтобы засечь чтото на огромном расстоянии, но и засечь ты сможешь понятное дело только если объект находится в твоём луче.

по этому для космоса лучше работает радар не на радиоволнах а на оптических, инфракрасных и всяких таких волнах (тоесть телескоп), тем более что удобно что в роли излучающей антенны мы имеем солнце с его просто ебической мощностью. так что если уж искать то телескопом. хотя если чисто принципиально рассуждать то радар и телескоп это примерно одно и то же, тоесть радар это тупо радиотелескоп но который не просто сам слушает но и сам излучает
Аноним 23/12/17 Суб 03:00:33  366448
>>366438
Очевидно же.
"Двуступенчетая требушера кб мерлина."
мимо.
Аноним 23/12/17 Суб 03:47:00  366450
>>366431
>>366427
>>366444
Спасибо.
Аноним 23/12/17 Суб 14:03:12  366523
Лучше бы на мой вопрос >>366212 ответили чем спорить с котопультером
Аноним 23/12/17 Суб 14:04:23  366524
>>366523
У тебя геймплея нет. Сделай игру, чтоб интересно было следить за разработкой и помогать.
Аноним 23/12/17 Суб 15:01:31  366547
>>366524
Из гемплея планируется только выращивание биомов и то вряд ли
Аноним 23/12/17 Суб 16:50:21  366563
До сих пор не верится, что я всё это время думал, что тень на луне это от Земли, а не её собственная.
Аноним 23/12/17 Суб 16:53:09  366565
>>366563
Ты, мягко говоря, не один такой. Хули удивляться, если астрономия из школ исчезла еще в начале 90-х?
Аноним 23/12/17 Суб 16:54:17  366566
15126445039730.jpg (337Кб, 1200x900)
>>366563
Но ведь тень от Земли тоже бывает
Аноним 23/12/17 Суб 16:54:38  366567
>>366563
Ничего страшного.
Есть дауны, неиронично думающие, что земля плоская.

>>366565
Ээ, а у нас она появилась в начале нулевых, а до этого я к этой же училке на факультатив по астрономии. Даже ей Red Shift 3 подогнал (свежак тогда)
Аноним 23/12/17 Суб 16:55:46  366568
>>366565
Так это по физике же вроде преподают.
Аноним 23/12/17 Суб 17:03:08  366569
Какова функция стартового стола?
Почему так дорого залить основание цементом и газоотводную трубку присобачить, а в случае отказа просто отреставрировать цементом и запускать дальше?
Это цемент дорогой или нынешние ракеты так хуёвы, что летают только под идеальными 90 градусов?
Аноним 23/12/17 Суб 17:04:27  366570
>>366566
Бывает во время затмений - одно, анон говорит про фазы Луны и вроде бы это очевидно, и нахуй ты вперся? Ты дохуя капитан очевидность, или просто из тех ебанатов, которые разевают еблет по любому поводу, потому что просто не в состоянии
заткнуться, или промолчать? Тебя в школе не пиздили за это?

>>366568
>Так это по физике же вроде преподают.
Где как, где вскользь упоминают, где не преподают совсем. У нас астрономии вообще не касались ни разу.

>>366567
>появилась в начале нулевых
В отдельных школах она и не заканчивалась, как дополнительный предмет, или факультатив. Но в обязательной программе ее не было. Так что в подавляющем большинстве случаев астрономии не было.
Аноним 23/12/17 Суб 17:05:30  366571
>>366569
>Какова функция стартового стола?
Держать ракету и подкармливать ее соляркой с воздухом и не быть распидорашенным при старте упомянутой ракеты.
>Почему так дорого залить основание цементом и газоотводную трубку присобачить, а в случае отказа просто отреставрировать цементом и запускать дальше?
Нужен хороший сопромат, просто зашпаклевав щель ты не сделаешь дырявый стол прочнее, он может зафейлить.
Потому грамотно заливают бетониум. Тут как с гидроэлектростанциями, абы как не построишь.
>Это цемент дорогой или нынешние ракеты так хуёвы, что летают только под идеальными 90 градусов?
Это конструкция с повышенными требованиями к надежности и перегрузкам и вибрациям от ракетного пердежа.
Аноним 23/12/17 Суб 17:07:10  366572
>>366570
>Бывает во время затмений - одно, анон говорит про фазы Луны и вроде бы это очевидно, и нахуй ты вперся? Ты дохуя капитан очевидность, или просто из тех ебанатов, которые разевают еблет по любому поводу, потому что просто не в состоянии
>заткнуться, или промолчать? Тебя в школе не пиздили за это?
Нихуя ты свахтёрился. Ну написал он и написал, хорошую картинку прикрепил, зачем триггериться?
>Но в обязательной программе ее не было.
У нас была, мне повезло.
>Так что в подавляющем большинстве случаев астрономии не было.
Это печально. Что хуже так это всякая параша на ютьюбе или хрентв которая пиздит про пришельцев, плоскую землю и "ряяя амеры не были на луне".
Аноним 23/12/17 Суб 17:17:40  366575
>>363848 (OP)
Поясните за космический лифт. Говорят что для него нужны крепкие и легкие тросы, которых сейчас нет. Но что мешает уже сейчас сделать цепь из аэростатов на разной высоте(в идеале еще и конусом, чтоб не сносило ветром), это же надежней и несуществующих тросов не надо.
Аноним 23/12/17 Суб 17:19:48  366576
>>366575
То, что твои аэростаты поддержат первые 10 километров лифта.
Остальные тридцать тысяч километров вне атмосферы чем держать собрался?
Аноним 23/12/17 Суб 17:20:36  366577
Почему космический мусор и метеориты не врезаются в спутники и станции на орбите?
Аноним 23/12/17 Суб 17:21:35  366578
>>366577
Потому, что орбита большая, а мусор маленький, потому попасть сложно.
Аноним 23/12/17 Суб 17:31:30  366579
>>366577
Вселенная больше, чем ты можешь вообразить. А если сможешь - сойдёшь с ума.
Аноним 23/12/17 Суб 17:35:21  366580
>>366576
Ну там уже поидее центростремительное ускорение присутствует, да и гравитация не такая сильная.

Аноним 23/12/17 Суб 17:36:54  366581
1507144482436.jpg (79Кб, 766x960)
>>366580
Я понимаю, что платина треда тупых вопросов и отличный тупой вопрос, но всё же руки опускаются.
Нет, сраные аэростаты погоды не сделают вообще, большая часть лифта весит дохуища и находится за пределами атмосферы.
Аноним 23/12/17 Суб 17:39:46  366582
>>363848 (OP)
Алсо, еще вопросы по теме аэростатов:
- насколько реально сделать летающий остров, состоящий из отсеков с гелием или водородом?
- возможна ли оболочка, полностью не пропускающая водород или гелий?
- возможна ли оболочка, полностью не пропускающая кислород, азот и углекислый газ(воздух короче)?
- возможно ли сделать летающую пену - что-то вроде пенопласта, но заполненного водородом? А из металла?

Аноним 23/12/17 Суб 17:42:21  366583
>>366582
>- насколько реально сделать летающий остров, состоящий из отсеков с гелием или водородом?
Ты только что дирижабль. Только большой.
>- возможна ли оболочка, полностью не пропускающая водород или гелий?
Нет.
>- возможна ли оболочка, полностью не пропускающая кислород, азот и углекислый газ(воздух короче)?
Полностью - нет. Но в пределах погрешности - да.
>- возможно ли сделать летающую пену - что-то вроде пенопласта, но заполненного водородом? А из металла?
Да. https://www.youtube.com/watch?v=rN-mfQsno2g
Из металла: ты только что опять дирижабль. На этот раз осне маленький.
Аноним 23/12/17 Суб 17:47:01  366584
>>366582
>- насколько реально сделать летающий остров, состоящий из отсеков с гелием или водородом?

Малореально.

>возможна ли оболочка, полностью не пропускающая водород

Легко.

>или гелий?

Реально, но посложнее.

>- возможна ли оболочка, полностью не пропускающая кислород, азот и углекислый газ(воздух короче)?

Легко.

> возможно ли сделать летающую пену - что-то вроде пенопласта, но заполненного водородом? А из металла?

Запросто, только оно будет много тяжелее воздуха.
Аноним 23/12/17 Суб 17:47:40  366585
>>366583
>Ты только что дирижабль.
Да, только Ну очень большой и статичный - то есть не опускающийся на землю, по крайней мере могущий в очень продолжительные полеты.

>Видео
А по-тверже и долговечней нет?
>На этот раз осне маленький.
Насколько маленький? Допустим из лития(он кстати хорошо еще и водород впитывает)?
Аноним 23/12/17 Суб 17:50:29  366587
>>366584
>Малореально.
Почему?

>Легко.
Можно пример подобного материала? Насколько толстым он должен быть и насколько хорошо не пропускает?

>Реально, но посложнее.
Почему? Гелий же вроде больше водорода, не?.

>Запросто, только оно будет много тяжелее воздуха.
Ну тогда она не будет летающей, лол.
Аноним 23/12/17 Суб 18:04:40  366590
image.png (3297Кб, 1920x1080)
>>366584
>
>>возможна ли оболочка, полностью не пропускающая водород
>Легко.
>>или гелий?
>Реально, но посложнее.
Наоборот, наркоман, гелий проще удержать, а водород травит только в путь.

>>366585
>Да, только Ну очень большой и статичный - то есть не опускающийся на землю, по крайней мере могущий в очень продолжительные полеты.
Технически можно, но это троллейбус из хлеба.

>А по-тверже и долговечней нет?
Picrelated.

>Насколько маленький? Допустим из лития(он кстати хорошо еще и водород впитывает)?
Зуй хнает, есть аэрогель, можно водородом напитать, но он тяжелее воздуха выйдет все равно по идее.
Чем меньше объект тем меньше он вместит водорода на единицу своей массы из-за того, что объем растет кубически а площадь поверхности (и следовательно масса этой поверхности) квадратически.

>>366587
>Почему?
Потому, что не нужно.

>Можно пример подобного материала? Насколько толстым он должен быть и насколько хорошо не пропускает?
Идеально отполированный алмаз может относительно хорошо водород удерживать, в нем и пытались металлический водород делать.

>Почему? Гелий же вроде больше водорода, не?.
Так и есть, он спутал, наверное.

>Ну тогда она не будет летающей, лол.
Можно налить в аквариум гексафторид серы...
Аноним 23/12/17 Суб 18:20:15  366594
>>366575
Троссоблядки должны страдать.
Аноним 23/12/17 Суб 18:28:22  366598
>>366578
Но мусора там дохрегище, а ещё больше метеоритов влетает.
>>366579
Бесконечная же.
Аноним 23/12/17 Суб 18:30:21  366599
>>366598
Не настолько много. Если разложить весь космусор на поверхности земли ты будешь очень долго блуждать пока не набредешь на кусок обтекателя или другую хрень. Космос большой. Мусора мало.
Аноним 23/12/17 Суб 18:36:57  366600
>>366599
То есть ты хочешь сказать, что на орбите находятся просто раскручивая рулетку?
Космос большой, но не орбита + куча метеоритов.
Аноним 23/12/17 Суб 18:38:41  366601
>>366600
>То есть ты хочешь сказать, что на орбите находятся просто раскручивая рулетку?
Че?
>Космос большой, но не орбита + куча метеоритов.
Он ОГРОМНЫЙ. Колоссально. Даже сраная орбита сраной земляшки.
Запусти кербач и попробуй состыковаться на орбите, чтобы примерно почувствовать масштаб.
Аноним 23/12/17 Суб 18:41:42  366603
>>366601
Пребывание на орбите получается рулетка, не то что полёты в дальний космос.
Воздушное пространство тоже огромно, но всё равно птицы залетали в турбины. Не может быть такого, что за всё время ни один метеорит не сбил спутник, их тысячи за сутки влетают.
Аноним 23/12/17 Суб 18:46:52  366604
Почему Солнце жёлтое, светит жёлтым но свет при этом белый?
Аноним 23/12/17 Суб 18:51:34  366606
image.png (1640Кб, 1344x742)
image.png (47Кб, 2000x1333)
>>366603
>Пребывание на орбите получается рулетка, не то что полёты в дальний космос.
Нет. На низкой есть микрометеоритная защита, а объекты крупнее с которой она не справится трекаются все поголовно, плюс у них орбита быстро деградирует и они сгорают.
На высокой орбите ОСНЕ МНОГО ПРОСТРАНСТВА.
Птицы залетают в турбины зачастую у аэропортов что близ берегов, а там птицы гнездятся и залетают в турбины на взлете/посадке, на эшелоне птицы лютая редкость.
>Не может быть такого, что за всё время ни один метеорит не сбил спутник, их тысячи за сутки влетают.
Такого практически не случается из-за колоссальных масштабов. Спутники на низкой чудом столкнулись (Иридиум с космосом н лет назад).
Ты просто очень переоцениваешь количество говна на орбите и очень недооцениваешь масштабы космического пространства.
Тебе кажется, что это пикрелейтед 1
На деле пикрелейтед 2
Аноним 23/12/17 Суб 18:52:54  366607
>>366604
Потому, что помимо желтого еще остальные цвета есть и их не сильно меньше. Но их чуть меньше, поэтому желтое.
Цветовая температура, слышал?
Лампочки не покупал никогда?
Аноним 23/12/17 Суб 19:04:50  366610
>>366607
Не слышал. Что это значит, от чего зависит и как влияет?
Аноним 23/12/17 Суб 19:06:22  366611
image.png (314Кб, 840x368)
image.png (213Кб, 606x476)
>>366610
Просвещайся.
https://en.wikipedia.org/wiki/Color_temperature
Аноним 23/12/17 Суб 19:30:57  366615
>>366587
>Почему?

Потому что такой остров должен быть очень толстым, считай сам.

>Можно пример подобного материала? Насколько толстым он должен быть и насколько хорошо не пропускает?

Таких материалов дохуя. ПЭ, ПВХ, ПУ, ПЭТ.


>Почему? Гелий же вроде больше водорода, не?.

У гелия один атом в молекуле, у водорода два. У гелия нет электронов за пределами первой оболочки, он очень маленький.

катапультобог
Аноним 23/12/17 Суб 19:39:54  366616
>>366590
>Наоборот, наркоман, гелий проще удержать, а водород травит только в путь.

Ссу тебе в ебало.
Скорость его диффузии сквозь твёрдые материалы в три раза выше, чем у воздуха, и приблизительно на 65 % выше, чем у водорода.

https://ru.wikipedia.org/wiki/Гелий

катапультобог
Аноним 23/12/17 Суб 19:42:51  366617
>>366575
> Говорят что для него нужны крепкие и легкие тросы, которых сейчас нет. Но что мешает уже сейчас сделать цепь из аэростатов на разной высоте(в идеале еще и конусом, чтоб не сносило ветром), это же надежней и несуществующих тросов не надо.

С ростом высоты быстро падает плотность воздуха и растёт скорость ветра, то предельная высота всей этой конструкции около 15км. Такая конструкция даст к концу работы первой ступени такую же прибавку к энергии, как стартовая скорость в примерно 80 м/с. То есть запускать ракету катапультой дешевле.
Аноним 23/12/17 Суб 20:20:34  366620
>>366568
Оканчивал школу в нулевые, вроде не деревня, астрономии не было как предмета. Я вообще узнал о каком-то массовом преподавании в школе астрономии лишь пару лет назад, когда об этом заговорили власти. Все знания по "астрономии" легли еще в 1-3 классе. По физике ничего подобного не было, в различных учебниках были разделы по механике, энергии, оптике, магнетизму, электричество, молекулерные/атомные силы. Для меня пиздец как удивительно читать, что кто-то с учителем спектры звезд изучал и орбитальную механику. Охуеть
Аноним 23/12/17 Суб 20:33:08  366621
image.png (176Кб, 1582x796)
Это такой толстый троллинг или совсем всё плохо в стране?
Аноним 23/12/17 Суб 20:58:52  366622
>>366621
как ты можешь серьезно кого-то воспринимать у кого на форуме 27к сообщений? Причем на твоем скриншоте он щитпостит два раза подряд.
Аноним 23/12/17 Суб 21:25:58  366629
>>366621
Это где такие пациенты?
Аноним 23/12/17 Суб 21:27:17  366630
>>366621
Не, шизики всегда были. Щас с интернетами и РенТВ это просто более явно видно.
Аноним 23/12/17 Суб 21:37:17  366631
>>366611
Есть зелёные лампочки, а где зелёные и фиолетовые звёзды?
Аноним 23/12/17 Суб 21:40:59  366632
>>366631
Нигде, их не существует.
Аноним 23/12/17 Суб 21:42:43  366633
>>366632
Почему? Разве они не должны повторять спектр радуги?

>Главную роль играет поверхностная температура. Солнечная достигает 6000 Кельвинов. На самом деле, его цвет белый, но земной наблюдатель воспринимает как желтый из-за прохождения света сквозь атмосферный фильтр.
То есть в космосе Солнце белое?
Аноним 23/12/17 Суб 21:46:00  366634
>>366633
>Разве они не должны повторять спектр радуги?
Нет. Вот спектр - >>366611
Червоний, жовтий, білий, блакитний. Всё.
>То есть в космосе Солнце белое?
Оно и так белое. Выжигающе сетчатку белое.
Аноним 23/12/17 Суб 21:50:19  366637
>>366634
>Червоний, жовтий, білий, блакитний. Всё.
Но ведь свет распадается на радужный спектр, почему в звёздах нет такого же отображения?
>Оно и так белое. Выжигающе сетчатку белое.
Тогда откуда берётся желтизна и прочие цвета?
Аноним 23/12/17 Суб 21:54:44  366639
image.png (225Кб, 660x440)
>>366637
>Но ведь свет распадается на радужный спектр, почему в звёздах нет такого же отображения?
Есть. Их свет тоже распадается на радугу.
>Тогда откуда берётся желтизна и прочие цвета?
Какая еще желтизна? Оно просто чуть желтее, чем просто белое из-за температуры. Было бы холоднее было бы еще желтее или краснее. Было бы горячее было бы голубым. См на цветовую температуру еще раз до просветления >>366611
Аноним 23/12/17 Суб 22:36:46  366644
>>366639
Тогда где зелёный и фиолетовые?
>Оно просто чуть желтее, чем просто белое из-за температуры
Летом всё залито жёлтым.
>Было бы горячее было бы голубым
Но ведь это холодный цвет, почему сначала идёт белый, а потом голубой? Метал раскаляется от красного к белому без голубого.
Аноним 23/12/17 Суб 22:39:20  366645
>>366644
>Тогда где зелёный и фиолетовые?
>>366639
>См на цветовую температуру еще раз до просветления >>366611
Ещё раз смотри. Не существует такой температуры чтоб светиться зеленым.

>Летом всё залито жёлтым.
Нет.

>Но ведь это холодный цвет
Нет.
>почему сначала идёт белый, а потом голубой?
Потому, что голубой горячее.
>Метал раскаляется от красного к белому без голубого.
Потому, что чтобы светиться голубым надо иметь такую температуру при которой металл уже не твердый, блядь, он плазма нахуй.
Аноним 23/12/17 Суб 22:45:03  366646
Аноны, вопрос: а кроме Celestia 2000 программ, предназначенных для профессиональной работы со звездными каталогами, нет?
Аноним 23/12/17 Суб 22:45:18  366647
>>366645
>Ещё раз смотри. Не существует такой температуры чтоб светиться зеленым.
Как так, спектр есть, а температуры нет?
>Нет.
На полюсе живёшь?
>Потому, что чтобы светиться голубым надо иметь такую температуру при которой металл уже не твердый, блядь, он плазма нахуй.
Как голубой может стоять над белым, если белый ярче? По яркости идут красный, оранжевый, жёлтый, белый.
Аноним 23/12/17 Суб 22:46:30  366648
>>366577
На самом деле тебе не сказали, что редко, оче редко, но бывает такое. Никто ничего не скрывает. Ну или по крайней мере скрывает не все на тему.
Аноним 23/12/17 Суб 22:48:00  366649
>>366646
И вдогонку: где ее можно скачать кроме торрента с одним дохлым сидом?
Аноним 23/12/17 Суб 22:48:52  366650
image.png (53Кб, 665x153)
image.png (926Кб, 1430x496)
image.png (587Кб, 1200x922)
image.png (170Кб, 640x320)
>>366647
>Как так, спектр есть, а температуры нет?
Вот так просто. Нет температуры для зеленого.

>На полюсе живёшь?
На Земле, а не в мультике. Солнце белое и светит белым. Желтеет/краснеет к закату ближе.
>По яркости идут красный, оранжевый, жёлтый, белый.
А потом голубой.
На ещё картинок, может поможет.
Аноним 23/12/17 Суб 22:50:54  366651
>>366646
Я в этом не особо разбираюсь, но может Stellarium подойдёт
Аноним 23/12/17 Суб 22:55:29  366652
>>366650
>Вот так просто. Нет температуры для зеленого.
Это не просто. Почему в свете зелёный есть, но цвета звёзд такого не бывает? Это противоречит законам физики.
>Солнце белое и светит белым. Желтеет/краснеет к закату ближе.
Летом в безоблачный день всё жёлтое, зимой тоже бывает. Белое только с облаками.
>А потом голубой.
>На ещё картинок, может поможет.
Не сильно помогает. Ну вижу я, что голубой, но откуда он там, если белый ярче? Это же несоответствие температуры цвету.
Аноним 23/12/17 Суб 22:55:43  366653
>>366651
Не, увы, это обычный планетарий. Celestia 2000 вроде как работает напрямую с каталогами Hipparcos и Tycho, может выводить данные по нужным критериям (я хз, я даже скачать ее не могу)
Аноним 23/12/17 Суб 22:58:49  366654
>>366652
>Это противоречит законам физики
Каким образом? Не, паринь, если тебе очень хочется фиолетовую звезду для своего космооперного высера, то можешь придумать сферу Дайсона, но в виде сплошного светофильтра. Дескать гламурные обитатели далекой системы запилили себе такую, шоб красива со стороны смотрелось.
Аноним 23/12/17 Суб 23:03:22  366655
image.png (111Кб, 1520x1080)
Blackbodyvisibl[...].gif (90Кб, 300x216)
>>366652
>Это противоречит законам физики.
Чьим? Твоим? Учи человеческие, им это не противоречит.
>Почему в свете зелёный есть, но цвета звёзд такого не бывает?
Потому, что излучение нагретых тел это не прогулка по радуге. Может гифка поможет?

>Летом в безоблачный день всё жёлтое
Попробуй баланс белого настроить, у тебя пиздец какой-то с цветопередачей, видать.

>Не сильно помогает. Ну вижу я, что голубой, но откуда он там, если белый ярче? Это же несоответствие температуры цвету.
Смотри, пока не поможет. Вот гифка может поможет.
Аноним 23/12/17 Суб 23:14:42  366657
>>366655
По твоей картинке красный идёт до жёлтого, с жёлтого в розовый перескакивает, а с розового в голубой. И где розовые звёзды?
>Потому, что излучение нагретых тел это не прогулка по радуге. Может гифка поможет?
Что там за буквы и какая взаимосвязь движения кривой с цветами снизу? Не все цвета из неё отображаются.
Аноним 23/12/17 Суб 23:17:45  366658
>>366654
Причём тут со стороны, мы говорим о правильном отображении спектра у звёзд. И если белый ярче голубого, то почему голубой горячее? Это как если бы после жёлтого или белого шёл внезапно красный.
Аноним 23/12/17 Суб 23:21:44  366660
wtf did i just [...].jpg (46Кб, 554x312)
>>366657
>розовый
Ты может просто дальтоник? Это обьяснит пожелтение зрения при нормальном солнечном свете и то, что ты видишь розовый где его нет.
>какая взаимосвязь движения кривой с цветами снизу
ПРЯМАЯ. На квадратике суммарный цвет, кривая показывает интенсивность цветов черного тела в зависимости от температуры. Всё, что нужно знать.

>Не все цвета из неё отображаются.
Они и не должны, тебе никто радугу не обещал, это ты себе нафантазировал, что по всем цветам радуги будут цвета.

>>366658
>И если белый ярче голубого
Нет. Яркость может быть любой, цвет не может быть ярче другого цвета, яркость определяется интенсивностью источника.
>почему голубой горячее
Потому, что чем горячее, тем больше в сторону УФ смещается спектр, блядь.
Аноним 23/12/17 Суб 23:28:39  366663
>>366660
>Ты может просто дальтоник? Это обьяснит пожелтение зрения при нормальном солнечном свете и то, что ты видишь розовый где его нет.
Или ты не улавливаешь оттенки.
>ПРЯМАЯ. На квадратике суммарный цвет, кривая показывает интенсивность цветов черного тела в зависимости от температуры. Всё, что нужно знать.
Если квадратик убрать ты мне по кривой назовёшь цвет, который она отображает?
>Они и не должны, тебе никто радугу не обещал, это ты себе нафантазировал, что по всем цветам радуги будут цвета.
Тогда почему сам свет на радугу раскладывается, а если предмет нагревать, то там он не светится?
>Нет. Яркость может быть любой, цвет не может быть ярче другого цвета, яркость определяется интенсивностью источника.
Всмотрись в квадратик на гифке и скажи честно, какой из них самый яркий.
>Потому, что чем горячее, тем больше в сторону УФ смещается спектр, блядь.
То есть самая горячая звезда будет фиолетовая. А какой будет звезда ещё горячее?
Аноним 23/12/17 Суб 23:33:00  366664
>>366663
>Если квадратик убрать ты мне по кривой назовёшь цвет, который она отображает?
Примерно почувствую только.
>Тогда почему сам свет на радугу раскладывается, а если предмет нагревать, то там он не светится?
Чеблядь? Перефразируй осмысленно. Может поймешь, где обосрался.
>Всмотрись в квадратик на гифке и скажи честно, какой из них самый яркий.
Вообще похуй, это цвета, у них нет яркости. Я могу севшим фонариком белым посветить на стену, а потом красной лазерной указкой в глаз, красный будет ярче белого, охуеть.
Яркость никакого значения не имеет. Есть цветовая температура и она выше у голубого и ниже у красного, то самое что на тыще картинок выше и показано.
>То есть самая горячая звезда будет фиолетовая. А какой будет звезда ещё горячее?
Двигайся дальше в ЭМ спектре. Пик излучения будет приходиться на жесткий ультрафиолет. Но в видимом спектре тоже будет излучение.
Аноним 23/12/17 Суб 23:34:23  366665
Блядь, астрофизика просто ебанутая, как это говно вообще понимать можно, оно же антилогичное. И самый пиздец в том, что никто этого не замечает кроме единиц таких как я. Мир делится на совсем тупорылых, которым похуй, и на задротов учёных, которых волнуют только циферки.
Аноним 23/12/17 Суб 23:38:12  366666
fukken lold.png (230Кб, 640x400)
>>366665
>Блядь, астрофизика просто ебанутая, как это говно вообще понимать можно, оно же антилогичное. И самый пиздец в том, что никто этого не замечает кроме единиц таких как я.
Нет, ну что ты! Есть полно таких как ты: >>223626 (OP)
https://2ch.hk/zog/

Ну и мог бы уточнить, что тебе "антилогично". Скорей всего таблетки не принял и логика поехала по пизде, либо просто умственно отсталый ретард не могущий в логику.
Аноним 23/12/17 Суб 23:39:53  366667
15140594028800.jpg (18Кб, 1520x1080)
>>366663
>Или ты не улавливаешь оттенки.
Анон, не хочу тебя расстраивать, но похоже у тебя действительно дальтонизм. На той пикче розовый - самый крайний справа. Между жёлтым и голубым - зелёный. Моя пикча какого цвета?
мимокрокодил
Аноним 23/12/17 Суб 23:45:48  366670
>>366664
>Примерно почувствую только.
Жопой?
>Чеблядь? Перефразируй осмысленно.
Куда исчезает зелёный?
>Вообще похуй, это цвета, у них нет яркости. Я могу севшим фонариком белым посветить на стену, а потом красной лазерной указкой в глаз, красный будет ярче белого, охуеть.
Я больше ничего говорить не буду, ты просто сейчас возьмёшь и сравнишь муон с футабой и макабой. А потом выключишь свет и повторишь. И всё при одной и той же яркости монитора.
>Двигайся дальше в ЭМ спектре. Пик излучения будет приходиться на жесткий ультрафиолет. Но в видимом спектре тоже будет излучение.
Если нагревать бесконечно, то как будет меняться?
Аноним 23/12/17 Суб 23:48:24  366671
>>366670
>>Примерно почувствую только.
>Жопой?
Да. Я пока не надрочился суммировать в голове спектр по интенсивности. РГБ могу примерно прикидывать, а спектр нет.
>Куда исчезает зелёный?
Никуда он не исчезает, он светит вместе со всеми остальными цветами.
>Если нагревать бесконечно, то как будет меняться?
Дальше в синюю сторону, очевидно же.
Аноним 23/12/17 Суб 23:50:08  366672
>>366667
Иссиня оранжевый.
Аноним 23/12/17 Суб 23:52:49  366674
>>366563
Лол, пиздец и впрямь
Аноним 23/12/17 Суб 23:55:32  366677
>>366671
>Никуда он не исчезает, он светит вместе со всеми остальными цветами.
Тогда почему нет такой звёзды, в которой бы зелёный выделялся?
>Дальше в синюю сторону, очевидно же.
Фиолетовый это конечный цвет для любой температуры выше?
Аноним 23/12/17 Суб 23:56:32  366678
>>366579
Страшновато за свою ранимую психику чёт стало
Аноним 23/12/17 Суб 23:58:37  366679
sBv1mGt.jpg (5Кб, 216x233)
>>366677
Потому, что так работает цветовая температура и наши глаза.
Когда пик приходится на зеленую часть видимого спектра остальные цвета видимого спектра тоже высокоинтенсивны давая в сумме белый цвет видимый глазами.

Уф, блин. Ты не хочешь ICQ померить? Это всё было изложено в статье про цветовую температуру.
Аноним 24/12/17 Вск 00:00:11  366680
>>366672
А там зелёный.
Аноним 24/12/17 Вск 00:03:25  366683
>>366679
Раз ты такой умный, скажи, как зависит ЯРКОСТЬ света от температуры? Как и излучение, пропорционально температуре в 4ой степени?
неон
Аноним 24/12/17 Вск 00:04:48  366684
>>366683
Никак не зависит.
Ты можешь взять 5000К лампочку на 3 ватта, а можешь на 30 ватт. Температура та же, яркость разная.
Аноним 24/12/17 Вск 00:21:38  366689
>>366646
Бамп. Буду заглядывать каждый день, может кто ответит. Пока нашел NOVAS и Starlink, но не уверен, что это то, что нужно, у меня английский хреновый.
Аноним 24/12/17 Вск 01:22:28  366696
>>366604
Кек, ты решил затролить спейсач таким старым и непростым вопросом?

http://dxdy.ru/topic58838.html

мимо
Аноним 24/12/17 Вск 01:47:25  366698
>>366679
>Когда пик приходится на зеленую часть видимого спектра остальные цвета видимого спектра тоже высокоинтенсивны давая в сумме белый цвет видимый глазами.
Ну вот это уже похоже на объяснение. Но почему именно при зелёном, а при других такого нет? Что за статья?
Аноним 24/12/17 Вск 02:03:15  366701
Наткнулся на это видео. Солнце получается вообще невидимое для глаз в космосе?
https://www.youtube.com/watch?v=NHWdd_GZ-g4
Аноним 24/12/17 Вск 02:18:52  366702
sNP1nNnFfhY.jpg (566Кб, 2560x1700)
>>366701
Этот чувак не астронавт, никогда не был в космосе, чтобы так уверенно поучать людей, не имеет образования или степени, и постит на каком-то подозрительном сайте под названием free energy
Астронавты и космонавты, которые на самом деле были в космосе никогда не говорили такой чуши и прекрасно видят Солнце. Смотреть на него без фильтров сложно, но ты знаешь где оно, а с фильтрами наверное можно.
Аноним 24/12/17 Вск 09:23:23  366715
>>366698
>Ну вот это уже похоже на объяснение.
Это, блин, видно по графику.
>>366698
>Что за статья?
>>366679
>Это всё было изложено в статье про цветовую температуру.
>статье про цветовую температуру
>Что за статья?
Ну не знаю, эта? https://en.wikipedia.org/wiki/Down_syndrome
А, нет, вот эта же https://en.wikipedia.org/wiki/Color_temperature
В которой в названии присутствуют слова "цветовая температура". Наверное, она.

>Но почему именно при зелёном, а при других такого нет?
Потому, что вселенная так работает, что при цветовой температуре солнца максимальная интенсивность приходится на видимую часть спектра давая в сумме то, что мы называем белым цветом, а мы, мясные мешки развились под солнцем и глаза большинства мясных мешков эволюционировали чтобы видеть оптимально оптическую часть спектра.
Это не вселенная охуела и спектр какой-то ебанутый нарисовала чтоб ты удивлялся, это мы видим оптимально видимую часть, притом РГБ рецепторы так развиты чтобы видеть в сумме белый.
Аноним 24/12/17 Вск 12:02:49  366733
>>366684
>Температура та же, яркость разная.
От чего тогда яркость источника свете зависит?
неон
Аноним 24/12/17 Вск 12:07:40  366734
>>366733
От мощности, вестимо.
Я там слукавил, конечно же. Когда речь про светодиодные лампы идет там не работает формула для черного тела, у диодных ламп синий светодиод и желтый фосфоресцирующий слой в сумме дающий белый разной температуры (зависит от желтой хрени).
В случае с температурным излучением (от нагрева как в лампе накаливания) яркость зависит от температуры (мощности), размера. Та же вольфрамовая нить в зависимости от размера может светить по-разному примерно с той же температурой - вспомни лампочку для фонарика и 100Вт лампочку для гостиной.
Если дать больше вольтаж на лампочку (и не сжечь ее при этом) она станет нагреваться сильнее, температура повысится, повысится яркость и цветовая температура слегка в белый уйдет.
Синим лампы накаливания не светят, при такой температуре вольфрам плавится.
Голубое свечение достигается либо в плазме, либо иными способами не связанными с нагревом (форсированное излучение как в газоразрядных лампах или светодиодах).
Аноним 24/12/17 Вск 12:45:14  366743
>>366734
>яркость зависит от температуры
Пропорционально температуре в четвертой степени, как и тепловое излучение?
Аноним 24/12/17 Вск 12:48:03  366746
>>366743
Мы про какую яркость щас?
https://ru.wikipedia.org/wiki/Сила_света
https://ru.wikipedia.org/wiki/Яркость
https://ru.wikipedia.org/wiki/Освещённость
Аноним 24/12/17 Вск 12:51:26  366749
>>366746
Ну хуй знает, первая м.б. У меня лава из вулкана вытекает и светится. Хочу яркость света установить реалистичную.
Аноним 24/12/17 Вск 13:00:02  366753
>>366749
https://www.quora.com/What-Is-The-Relation-Between-Intensity-of-Light-and-Temperature
Может это интересует?
Аноним 24/12/17 Вск 13:01:08  366754
>>366684

Яркость не зависит от мощности, гугол в помощь.

катапультобог
Аноним 24/12/17 Вск 13:06:23  366757
>>363848 (OP)
Мне кажется или у спейс шаттла дюзы стоят от Сатурна5?
Аноним 24/12/17 Вск 13:09:26  366758
>>366754
>Яркость не зависит от мощности
Спешите видеть, человек никогда не знавший лампы накаливания!
Аноним 24/12/17 Вск 13:12:14  366759
>>366754
>катапультобог
Ты сопромат посчитал уже, или по-прежнему флакон хеви собираешься лифтовыми тросами запускать, лол?
Аноним 24/12/17 Вск 15:59:13  366779
dd3b2d9823dfc7c[...].jpg (270Кб, 600x378)
644770maindsc30[...].jpg (367Кб, 1000x1506)
>>366757
Тебе кажется. У них даже размеры разные, не говоря уже обо всём остальном. И пиши лучше сопла, у нас их дюзами не называют
Аноним 24/12/17 Вск 16:52:43  366788
3229A7F8-F6DB-4[...].jpeg (959Кб, 4288x2848)
Объясните мне одну очевидную вещь, ибо я ничего не понимаю. Между атмосферами планет - вакуум, то есть абсолютное беспросветное нихуя. Каким образом в этом нихуе какой-либо механический объект может менять свое направление? Самолет, создавая тягу, отталкивается от воздуха - а космический шаттл отталкивается от чего? В фантастических фильмах для этого корабли распыляют некую жидкость и таким образом вращаются. Но ведь для этого требуется сопротивление. Какое может быть сопротивление в вакууме, если вакуум - это нихуя? Объясните, я нихуя не понимаю.
Аноним 24/12/17 Вск 17:03:36  366790
>>366788
Закон сохранения импульса ещё не проходили?
Аноним 24/12/17 Вск 17:07:00  366792
Поясните за свет.
Если в X световых лет погаснет звезда, то мы сразу узнаем об этом, или остаточный свет уже без источника будет идти к нам?
Аноним 24/12/17 Вск 17:07:55  366793
>>366788
> космический шаттл отталкивается от чего?
От реактивной струи из двигателя, а струя с такой же силой отталкивается от шаттла. Закон сохранения импульса жи есть
24/12/17 Вск 17:11:00  366794
1005486428.jpg (134Кб, 538x700)
>>366788
>Самолет, создавая тягу, отталкивается от воздуха - а космический шаттл отталкивается от чего?
Аноним 24/12/17 Вск 17:13:46  366795
>>366792
второе и компот
Аноним 24/12/17 Вск 17:26:43  366797
>>366795
А как мы можем видеть этот свет, если он не доходит до наших глаз? Если фонарик может досветить только на 10 метров, как мы можем видеть его свет с расстояния 100 метров?
Аноним 24/12/17 Вск 17:30:39  366798
>>366797
>если он не доходит до наших глаз
Он доходит. Просто ему нужно время.
Аноним 24/12/17 Вск 17:45:54  366799
>>366797
>А как мы можем видеть этот свет, если он не доходит до наших глаз?

А мы и не видим. Рептилии уже может украли солнце 8 минут как, а ты и не знаешь. Потому что даже гравитационные волны не дошли до тебя от исчезнувшего солнца, как и свет. (точнее, не дошло их отсутствие)
Аноним 24/12/17 Вск 17:48:57  366800
>>366799
Если фонарик не светит дальше 10 метров, как его лампочку можно видеть со 100 метров?
Аноним 24/12/17 Вск 17:50:44  366801
>>366800
> Если фонарик не светит дальше 10 метров
Это как? Куда фотоны денутся?
Аноним 24/12/17 Вск 17:53:11  366802
>>366801
Попробуй ручным фонариком осветить 500 метров.
Аноним 24/12/17 Вск 17:55:31  366803
>>366802
Катапультист, ты?
Рассеивание\поглощение
А так твои фотоны почти бесконечно летят
Аноним 24/12/17 Вск 17:56:01  366804
>>366802
А при чём тут звёзды?
Аноним 24/12/17 Вск 18:07:33  366805
>>366802
Ну а как тебе такой ответ - за 500 метров темной ночью без погодных капризов ты прекрасно увидишь горящий фонарик, если им кто-то посветит в твою сторону. А еще прикинь, отраженный солнечный свет от космических аппаратов, катающихся по орбите, видно с поверхности. Я понимаю, что ты имеешь ввиду, когда говоришь про разницу между десятью и пятистами метрами для фонарика. Но как ты это на свой вопрос натягиваешь - непонятно.
Аноним 24/12/17 Вск 18:12:07  366807
>>366689
Бамп
Аноним 24/12/17 Вск 18:42:49  366813
>>366802
Осветить получится не очень.
Но фонарик можно при должной технике узреть даже из ебеней - в оптический прицел снайперской винтовки или бинокль.
Так и со звездами - они нихуя не освещают землю (по одиночке если считать), но видно их прекрасно при ясном небе без засветки.
Аноним 24/12/17 Вск 19:50:54  366834
>>366813
И как, если свет не доходит до глаза?
Аноним 24/12/17 Вск 19:51:40  366835
>>366834
Где не доходит? Ты про че?
Попробуй веки разомкнуть, всё доходит.
Аноним 24/12/17 Вск 19:51:46  366836
>>366805
Солнечный свет это не свет фонарика, он доходит до земли.
Аноним 24/12/17 Вск 19:52:35  366837
>>366803
Но рассеянный свет фонарика невозможно увидеть с расстояния 20 метров.
Аноним 24/12/17 Вск 19:54:41  366839
>>366837
Возможно даже с километра.
Даже долбаную зажигалку видно.
Даже тлеющую сигарету.
Ты с дна морского капчуешь что ли?
У нас в прозрачной атмосфере (и тем более вакууме) свет свободно распространяется.
Аноним 24/12/17 Вск 19:56:07  366840
>>366839
И как это объясняется? Свет же не доходит до глаза на таком расстоянии.
Аноним 24/12/17 Вск 19:57:34  366841
>>366840
Найс тралишь))))00
Аноним 24/12/17 Вск 20:03:18  366843
>>366840
Мы, кажется, установили связь с другой Вселенной.
В этой свет распространяется бесконечно.
Аноним 24/12/17 Вск 20:09:36  366845
>>366843
Если бы свет распространялся бесконечно, то зажигалкой можно было бы досвечивать до Америки. Одну свечку поставил и везде вокруг светло. Но если происходит рассеивание, то получается что свет сечётся очень быстро и не доходит дальше, где темно.
Аноним 24/12/17 Вск 20:12:09  366846
>>366845
Нет. Досветить до Америки можно было бы, если бы источник света испускал бесконечное количество фотонов. А он испускает конечное. Которые распространяются бесконечно.
Так что при должной сноровке и оборудовании можно задетектить свет твоей зажигалки в Америке. Самая большая проблема будет отфильтровать от другого света, естественно.
Аноним 24/12/17 Вск 20:23:26  366848
>>366846
Речь о том, что до глаза свет не доходит уже на расстоянии 200 метров как и до остальных предметов, иначе бы их было видно в темноте.
Аноним 24/12/17 Вск 21:03:18  366854
>>366848
Ты путаешь освещённость со светом.
Аноним 24/12/17 Вск 21:08:03  366855
>>366848
>что до глаза свет не доходит уже на расстоянии 200 метров
Выражение имеет физическую ошибку - свет дойдет до глаза, способен ли глаз такую интенсивность зарегистрировать - большой вопрос.
Аноним 24/12/17 Вск 23:45:51  366886
>>366848
Чето у меня из окна 20 этажа видно свет домов в 4 километрах и самолетов в 8 километрах.
Обман зрения? Самолет в 200 метрах?
Аноним 25/12/17 Пнд 00:27:58  366893
Вопрос знатокам! Почему левое " плечо" Ориона горит красным светом? Оно всегда было таким? Я часто смотрел на него, но красного света не видел!
Аноним 25/12/17 Пнд 01:04:04  366902
.png (65Кб, 300x337)
>>366715
>Потому, что вселенная так работает, что при цветовой температуре солнца максимальная интенсивность приходится на видимую часть спектра давая в сумме то, что мы называем белым цветом, а мы, мясные мешки развились под солнцем и глаза большинства мясных мешков эволюционировали чтобы видеть оптимально оптическую часть спектра.
Что значит оптимальная оптическая часть спектра? Глаза видят красный, жёлтый и оранжевый, но почему-то не видят зелёный. на графике линия проходит вообще мимо в области белого вне зависимости от глаз. Нагревание вообще можно изменить таким образом, чтобы добиться других цветов?
Аноним 25/12/17 Пнд 01:54:25  366906
>>366893
Там Бетельгейзе, она всегда была красная. По крайней мере, на всем протяжении существования лысых обезьян на Земле.
Аноним 25/12/17 Пнд 02:09:25  366910
>>366906
Спасибо за ответ.
Аноним 25/12/17 Пнд 11:14:05  366942
>>366902
Где-то слышал манятеорию о том, что пока лысые макаки еще не были лысыми и сычевали на деревьях во всяких джунглях дохуя, то у них глаза эволюционировали для адаптации к зеленому свету больше, чем к другим цветам. Может поэтому зеленый и таким особняком держится.
Аноним 25/12/17 Пнд 11:50:05  366947
>>366759

А что плохого в лифтовых тросах? Я просто напоминаю, что для движения с ускорением 20м/с/с усилие на тросах в три раза больше, чем просто при удерживании покоящейся ракеты.

>>366758

Яркость это световой поток в телесный угол, при использовании рассеивателей яркость падает. Лампы накаливания (обычные, не галогенки, не ДРВ) бывают прозрачными и матовыми, у матовых яркость значительно ниже.

>>366902

Зелёный свет есть и длина волны, ему соответствующая -555нм. Раскалить тело до зелёного света нельзя, но получить его можно при помощи фильтров или призм.

катапультобог
Аноним 25/12/17 Пнд 12:02:46  366953
5 gen fighter.png (965Кб, 822x665)
>>366947
>катапультобог
Зачем ты подписываешься, тебя и так легко определить.

>>366947
>А что плохого в лифтовых тросах?
Их хуевая надежность.
Но ты можешь допустить их стопроцентную надежность и посчитать сколько тросов надо для подъема флакона, какая нагрузка будет на ферменные конструкции, сколько будет стоить сооружение ферм поддерживающих блоки.
Аноним 25/12/17 Пнд 12:13:11  366956
>>366953
>Их хуевая надежность.

Это твои маняфантазии, не более того.
Практика показывает, что тросы эксплуатируются на миллионах лифтов, которые совершают миллиарды рейсов в день и отказ из-за троса крайне редкое явление. Нагрузка, допустимая на канат, более чем в 10 раз меньше суммарной прочности всех проволок в нём.
http://www.metizorel.ru/kanat3077.html

>какая нагрузка будет на ферменные конструкции,

Нагрузка на сжатие в момент запуска-примерно 6-10 веса ракеты, то есть до 6 тысяч тонн силы.
Аноним 25/12/17 Пнд 12:14:19  366957
ПОМПАЖ7.png (20Кб, 964x598)
>>366953

Ах да, пикча отклеилась.
Аноним 25/12/17 Пнд 12:56:59  366968
Не уверен туда ли пишу, но всё же.
Накачал на KSP реалистичных модов и охуел. Движки можно запускать 1 раз и только на земле и нельзя регулировать тягу.
Это так и работает ИРЛ? Это что, заранее надо рассчитать сколько топлива залить надо, чтоб ровно первая ступень отработала, а потом вторая?
Это так и работает?
Аноним 25/12/17 Пнд 13:00:29  366969
image.png (170Кб, 293x800)
>>366968
Да. Некоторые движки могут в многоразовый запуск.
Шаттловские, например, вообще имели что-то вроде свечи зажигания в маленькой камере сгорания над основной. Мерлины и некоторые другие движки используют небольшой запас забористого компонента который с топливом гиперголичен и поджигается самостоятельно.
Большинство пихол же одноразовые и поджигаются на старте. Союзовские, например, поджигаются длинными большими "спичками" пикрелейтед.
Кербач, увы, переехал в веге, к слову.
Аноним 25/12/17 Пнд 13:03:38  366971
>>366969
Тяга тоже ниригулируется?
Аноним 25/12/17 Пнд 13:05:32  366972
>>366971
У первых ракет регулировалась примерно никак.
У современных дросселируется в пределах 70-105% от номинала.
У йобистых может разброс до 50% доходить АФАИР.
Дросселирование бывает, но это люто сложная вещь, т.к. надо обеспечить плавность потока топлива и окислителя при разном давлении и избегать гидроудара.
В общем куча нюансов, о которых наш Анон "ракетнедвижки это просто" Троллевич забывает.
Аноним 25/12/17 Пнд 13:09:45  366974
>>366972
>У йобистых может разброс до 50% доходить АФАИР
Дык у того же шуттла: 65-109%
Аноним 25/12/17 Пнд 18:23:50  367016
>>366753
>https://www.quora.com/What-Is-The-Relation-Between-Intensity-of-Light-and-Temperature
>Может это интересует?
Т.е. яркость таки пропорциональна температуре в 4ой степени и углу излучения. Спасибо. Хотя угол я учитывать не буду.
ПЕРЕКОТ Аноним 25/12/17 Пнд 19:31:30  367034
ПЕРЕКОТ

>>367033 (OP)

https://2ch.hk/spc/res/367033.html
Аноним # OP  25/12/17 Пнд 19:33:51  367035
>>367034
С нулевой не ушло же, я бы перекатил.
Могут удолить.
Аноним 25/12/17 Пнд 19:47:36  367046
>>367035
Сходи покури, новичок.
Аноним # OP  25/12/17 Пнд 19:48:51  367047
my sides 2.gif (817Кб, 400x398)
>>367046
>новичок
Ох, лоол.
Как скажешь, батя.
Аноним 25/12/17 Пнд 23:04:24  367097
matrix.jpg (444Кб, 1000x540)
>>366942
>лысые макаки еще не были лысыми и сычевали на деревьях во всяких джунглях дохуя
Всё гораздо проще и реалистчнее, чем это фольклор.
Аноним 26/12/17 Втр 16:19:12  367341
>>367097


Топ тредов
Избранное